04.07.2013 Views

Chapter 1 solutions - University of Minnesota

Chapter 1 solutions - University of Minnesota

Chapter 1 solutions - University of Minnesota

SHOW MORE
SHOW LESS

Create successful ePaper yourself

Turn your PDF publications into a flip-book with our unique Google optimized e-Paper software.

Organic Chemistry<br />

With a Biological Emphasis<br />

Solutions Manual<br />

Tim Soderberg<br />

<strong>University</strong> <strong>of</strong> <strong>Minnesota</strong>, Morris<br />

2010


In-chapter exercises<br />

E1.1: Mass numbers are:<br />

a) 31 P<br />

b) 32 P<br />

c) 37 Cl<br />

E1.2:<br />

E1.3:<br />

E1.4:<br />

a) a neutral magnesium atom: 1s 2 2s 2 2p 6 3s 2<br />

b) a Mg 2+ atom: 1s 2 2s 2 2p 6<br />

c) a neutral potassium atom: 1s 2 2s 2 2p 6 3s 2 3p 6 4s 1<br />

d) a K + ion: 1s 2 2s 2 2p 6 3s 2 3p 6<br />

<strong>Chapter</strong> 1<br />

<strong>Chapter</strong> 1 <strong>solutions</strong><br />

1


e) a Cl - ion: 1s 2 2s 2 2p 6 3s 2 3p 6<br />

E1.5:<br />

E1.6:<br />

E1.7:<br />

E1.8:<br />

2<br />

<strong>Chapter</strong> 1


E1.9:<br />

E1.10:<br />

E1.11:<br />

a) C 5H 12<br />

b) C 4H 10O<br />

<strong>Chapter</strong> 1<br />

3


c) C3H9N<br />

E1.12:<br />

a) 2-deoxycytidine: IHD = 5 (2 rings, 3 double bonds)<br />

b) histidine: IHD = 4 (2 rings, 2 double bonds)<br />

c) C9H10NOI: IHD = 5<br />

d) an ion with the molecular formula C6H11O7 - : IHD = 1<br />

E1.13:<br />

a) carboxylate, sulfide, aromatic, two amide groups (one <strong>of</strong> which is cyclic)<br />

b) tertiary alcohol, thioester<br />

c) carboxylate, ketone<br />

d) ether, primary amine, alkene<br />

E1.14:<br />

The following molecules fit the descriptions in the problem:<br />

4<br />

<strong>Chapter</strong> 1


E1.15:<br />

acetic acid: ethanoic acid<br />

chlor<strong>of</strong>orm: trichloromethane<br />

acetone: 2-propanone<br />

E1.16:<br />

fig3<br />

E1.17:<br />

E1.18:<br />

The bonding in chlor<strong>of</strong>orm consists <strong>of</strong> three <strong>of</strong> the sp 3 hybrid orbitals on the central<br />

carbon atom overlapping with 3p orbitals on clorine. The fourth bond is an overlap<br />

between the fourth sp 3 orbital on carbon and the 1s orbital <strong>of</strong> hydrogen.<br />

fig 3<br />

E1.19:<br />

<strong>Chapter</strong> 1<br />

5


6<br />

<strong>Chapter</strong> 1<br />

Both the carbon and the nitrogen atom in CH3NH2 are sp 3 -hybridized. The C-N σ bond<br />

is an overlap between two sp 3 orbitals.<br />

fig 4<br />

E1.20:<br />

fig 4<br />

E1.21:<br />

a) Csp 2 -Csp 2<br />

b) Csp 2 -Csp 3<br />

c) Csp 2 -Nsp 2<br />

d) Csp 3 -Nsp 3<br />

E1.22:<br />

The three bonds that are drawn as solid wedges should be drawn as lines – they are all<br />

coming out <strong>of</strong> planar, sp 2 -hybridized carbons and should be shown lying in the plane <strong>of</strong><br />

the page.<br />

E1.23:<br />

fig 4<br />

The carbon and nitrogen atoms are bond sp 2 -hybridized. The carbon-nitrogen double<br />

bond is composed <strong>of</strong> a σ bond formed from two sp 2 orbitals, and a π bond formed from<br />

the side-by-side overlap <strong>of</strong> two unhybridized 2p orbitals.<br />

E1.24:


a) The σ bond indicated by this arrow is formed by the overlap <strong>of</strong> an sp 2 orbital <strong>of</strong> a<br />

carbon atom and an sp 3 orbital <strong>of</strong> a carbon atom.<br />

b) The σ bond indicated by this arrow is formed by the overlap <strong>of</strong> an sp2 orbital <strong>of</strong> a<br />

carbon atom and an sp 2 orbital <strong>of</strong> a carbon atom.<br />

c) The σ bond indicated by this arrow is formed by the overlap <strong>of</strong> an sp2 orbital <strong>of</strong> a<br />

nitrogen atom and an s orbital <strong>of</strong> a hydrogen atom.<br />

d) The σ bond indicated by this arrow is formed by the overlap <strong>of</strong> an sp 2 orbital <strong>of</strong> a<br />

carbon atom and an sp 3 orbital <strong>of</strong> a carbon atom.<br />

d) The σ bond indicated by this arrow is formed by the overlap <strong>of</strong> an sp 3 orbital <strong>of</strong> a<br />

carbon atom and an sp 3 orbital <strong>of</strong> a carbon atom.<br />

End-<strong>of</strong>-chapter problems<br />

P1.1: (it will help to refer to color versions <strong>of</strong> the figures below, in the pdf file)<br />

fig 4<br />

<strong>Chapter</strong> 1<br />

7


fig 5<br />

P1.2:<br />

a) a lithium cation (Li + ): 1s 2<br />

b) a calcium cation (Ca 2+ ): 1s 2 2s 1 2p 6 3s 2 3p 6<br />

c) a iron cation in the ferric (Fe +3 ) state: 1s 2 2s 1 2p 6 3s 2 3p 6 4s 2 3d 3<br />

8<br />

<strong>Chapter</strong> 1<br />

P1.3: There are many possible correct answers to this problem – check your answer with<br />

your TA or instructor.<br />

P1.4:


<strong>Chapter</strong> 1<br />

fig 5<br />

Notice in doing this problem that it is much easier to determine the IHD from the<br />

structure rather than the molecular formula – just count rings and multiple bonds! But it<br />

would still be good practice to use the molecular formulas to calculate IHDs, then check<br />

that they match what you determined from the structures.<br />

P1.5: In the structures shown below:<br />

a-i) Describe the orbitals involved in the bonds indicated by the arrows.<br />

j) Fill in all formal charges<br />

k) Give the molecular formula for arginine<br />

a-i; j) see structures below<br />

9


fig 6<br />

k) the molecular formula <strong>of</strong> arginine is C 6H 15N 4O 2.<br />

P1.6:<br />

a) Csp 3 – Osp 3<br />

b) Csp 2 – Csp 3<br />

c) Csp 2 – Nsp 2<br />

d) Csp 2 – Csp 2<br />

e) Csp 3 – Csp 3<br />

f) Csp 2 – Csp 2<br />

g) Csp 3 - Csp 3<br />

h)Csp 2 – H 1s<br />

i) Csp 2 – Osp 2<br />

j) Csp 2 – Cl 3p<br />

k) Nsp 3 – H 1s<br />

10<br />

<strong>Chapter</strong> 1<br />

l) Compound 3 contains two aldehydes, compound one contains an ether, compound 2<br />

contains an amide, compound 3 contains a terminal alkene, and compound 4 contains a<br />

secondary amine.<br />

m) The molecular formula <strong>of</strong> compound 3 is C 10H 14O 2.<br />

P1.7:<br />

fig 6<br />

P1.8: Draw structures for four different amides with molecular formula C 3H 7NO.<br />

fig 6


P1.9: shortest e


12<br />

<strong>Chapter</strong> 1<br />

Notice that the central carbon is sp-hybridized, while the two end carbons are sp 2 -<br />

hybridized: the left one with the three sp 2 hybrid orbitals in the plane <strong>of</strong> the page, the<br />

right one with two if the three orbitals pointing into or out <strong>of</strong> the plane <strong>of</strong> the page.


In-chapter excercises<br />

<strong>Chapter</strong> 2<br />

<strong>Chapter</strong> 2 <strong>solutions</strong><br />

E2.1: Arrows point to the isolated double bonds. The other four double bonds are all part<br />

<strong>of</strong> a single conjugated system.<br />

fig 1E2.<br />

E2.2:<br />

13


E2.3:<br />

14<br />

fig 1<br />

fig 1<br />

The structure on the right above is the minor contributor by rules 5, 6, and 7: in this<br />

structure the carbon does not have a complete octet (rule 5), there is a separation <strong>of</strong><br />

charge (rule 6), and there are overall fewer bonds (rule 7).<br />

E2.4:<br />

fig 2<br />

E2.5:<br />

<strong>Chapter</strong> 2


fig 2<br />

E2.6<br />

fig 2<br />

E2.7:<br />

a)<br />

fig 3<br />

b) The conjugated π system in this carbocation is composed <strong>of</strong> seven p orbitals<br />

containing six delocalized π electrons.<br />

E2.8:<br />

<strong>Chapter</strong> 2<br />

15


E2.9:<br />

16<br />

fig 3<br />

c) The conjugated π system in structure a) is composed <strong>of</strong> seven p orbitals<br />

containing eight delocalized π electrons.<br />

<strong>Chapter</strong> 2<br />

Horizontal trend: A fluorine atom has one more proton in its nucleus than does oxygen,<br />

which in turn has one more proton than does nitrogen, and so on. An additional proton<br />

means an additional positive charge, so the (negatively charged) electrons are pulled in<br />

closer to the nucleus. This trend holds within a row <strong>of</strong> the periodic table, because the<br />

outermost electrons <strong>of</strong> all atoms in a row are all in the same energy ‘shell’.<br />

Vertical trend: Atoms furthur down a column have increasing numbers <strong>of</strong> electrons<br />

,which occupy energy ‘shells’ that are further away from the nucleus (eg. 3p for sulfur vs.<br />

2p for oxygen). In other words, the ‘electron cloud’ <strong>of</strong> sulfur is larger than that <strong>of</strong><br />

oxygen, meaning that the electrons are inherently further away from the nucleus in sulfur.<br />

Sulfur is less electronegative than oxygen, phosphorus less electronegative than nitrogen,<br />

and chlorine less electronegative than fluorine.<br />

E2.10: Molecules a, c, and d have dipole moments.<br />

E2.11:<br />

fig 3


E2.12:<br />

Vitamin C has many potential hydrogen-bonding sites, and thus is water-soluble.<br />

Vitamin B 3 has carboxylate and amine groups that are expected to be charged at<br />

physiological pH (we’ll learn more about this is chapter 6), thus it is water-soluble.<br />

<strong>Chapter</strong> 2<br />

Vitamin A has only one hydrogen-bonding group and a large hydrophobic component,<br />

thus it is fat-soluble.<br />

E2.13:<br />

Analine is a base and will accept a proton from hydrochloric acid. The resulting positive<br />

charge makes it water-soluble.<br />

Phenol does not act as a base in this context, and remains uncharged and water-insoluble.<br />

fig 3<br />

E2.14:<br />

Boiling points ( o C)<br />

benzene: 80<br />

benzaldehyde: 178<br />

phenol: 182<br />

benzoic acid: 249<br />

Benzoic acid has the highest boiling point because it has a very polar, hydrogen-bonding<br />

carboxylic acid group with two electronegative oxygens (benzoic acid is a crystalline<br />

solid at room temperature, the other three compounds are liquids).<br />

Phenol has a hydrogen-bonding hydroxyl group, but only one oxygen as opposed to two.<br />

Benzaldehyde has a polar carbonyl group, but because a carbonyl can only serve as a<br />

hydrogen bond accepter and not a donor, a benzaldehyde molecule cannot hydrogen bond<br />

to another benzaldehyde. Benzene is capable only <strong>of</strong> hydrophobic intramolecular<br />

interactions, and has the lowest boiling point.<br />

17


End-<strong>of</strong> chapter problems<br />

18<br />

<strong>Chapter</strong> 2<br />

P2.1: Both bonds in O 3 have a bond order <strong>of</strong> 1.5, and are the same length – the two<br />

resonance contributors shown below are <strong>of</strong> equal importance in illustrating the bonding in<br />

the molecule.<br />

fig 4<br />

P2.2:<br />

P2.3:


fig 4<br />

P2.4:<br />

fig 5<br />

P2.5:<br />

fig 5<br />

P2.6:<br />

<strong>Chapter</strong> 2<br />

19


fig 5<br />

20<br />

<strong>Chapter</strong> 2<br />

The He 2 molecule has four electrons to account for in the MO diagram: two are placed in<br />

the σ bonding orbital and two in the σ* antibonding orbital. Thus there is no net<br />

attractive force to hold the He 2 molecule together (in fact, there is a net repulsive force<br />

due to the relative energy level <strong>of</strong> the σ* molecular orbital).<br />

P2.7:<br />

fig 5,6<br />

P2.8:


fig 6<br />

P2.9:<br />

fig 7<br />

P2.10:<br />

<strong>Chapter</strong> 2<br />

21


fig 7<br />

P2.11:<br />

fig 7<br />

P2.12:<br />

P2.13:<br />

22<br />

<strong>Chapter</strong> 2


1= lowest bp<br />

fig 8<br />

Reasoning:<br />

<strong>Chapter</strong> 2<br />

a) 2 and 3 have two fluorines and are more polar than 1, so they have stronger<br />

intermolecular interactions. 3 has one more carbon than 2, and therefore stronger van der<br />

Waals interactions. 4 is capable <strong>of</strong> hydrogen bonding, so it has the strongest<br />

intermolecular interactions and the highest boiling point.<br />

b) 1 and 2 have only van der Waals interactions, but 2 has more carbons so these<br />

interactions are slightly stronger. 3 has a polar carbonyl group, and 4 is capable <strong>of</strong><br />

hydrogen bonding.<br />

c) 1 is not capable <strong>of</strong> hydrogen bonding. 2 and 3 both have hydrogen bonding groups,<br />

but 3 has one more carbon and therefore stronger overal van der Waals interactions.<br />

d) 1 has only van der Waals interactions. 2 has a polar thiol group, but 3 has a hydroxyl<br />

group which is capable <strong>of</strong> hydrogen bonding. 4 is a salt: the charge-charge interactions<br />

are very strong and lead to a very high boiling point.<br />

P2.14:<br />

a) The one on the right is more soluble (fewer hydrophobic carbons)<br />

23


) The one on the left is more soluble (ionic phosphate group)<br />

c) The one on the left is more soluble (fewer hydrophobic carbons)<br />

d) The one on the left is more soluble (capable <strong>of</strong> hydrogen bonding)<br />

e) The one on the right is more soluble (fewer hydrophobic carbons)<br />

24<br />

<strong>Chapter</strong> 2<br />

P2.15: Toluene has one more carbon than benzene, and therefore has stronger overal van<br />

der Waals interactions and a higher boiling point. The melting point trend, however, is a<br />

different story. Remember that melting involves a transition from an ordered solid state,<br />

where molecules are stacked together, to a disordered liquid state, where molecules move<br />

freely relative to one another. Benzene is a flat molecule and is able to stack very<br />

effeciently (like plates) in the solid phase. The methyl group <strong>of</strong> toluene prevents this<br />

efficient packing, and thus the overal intermolecular van der Waals interactions in the<br />

toluene solid phase are lower than those for benzene, and it requires less energy (heat) to<br />

break the tolune molecules apart.<br />

fig 8<br />

P2.16:<br />

fig 9<br />

P2.17: For each structure, the atoms are indicated (with a black dot) which could bear the<br />

corresponding negative formal charge, and one example <strong>of</strong> a resonance contributor is<br />

provided.


fig 9<br />

<strong>Chapter</strong> 2<br />

P2.18: For each structure, the atoms are indicated (with a black dot) which could bear the<br />

corresponding positive formal charge, and one example <strong>of</strong> a resonance contributor is<br />

provided.<br />

fig 9<br />

P2.19: For each structure, the atoms to which the positive charge can be delocalized are<br />

indicated with a black dot.<br />

25


In-chapter exercises<br />

E3.1:<br />

fig 1<br />

E3.2:<br />

fig 1<br />

E3.3:<br />

26<br />

<strong>Chapter</strong> 3<br />

<strong>Chapter</strong> 3 <strong>solutions</strong><br />

Mannose is a diastereomer <strong>of</strong> glucose: compare the major conformer shown below to the<br />

chair form <strong>of</strong> B-glucose (section 3.2C) and notice that they differ only in the<br />

configuration at C2.


<strong>Chapter</strong> 3<br />

fig 1<br />

In the major chair form <strong>of</strong> mannose, all substituents except the C2 hydroxyl are equitorial.<br />

E3.4.<br />

fig 1<br />

E3.5: Stereocenters are marked with a bold dot.<br />

fig 2<br />

E3.6: Fumarate, aspirin, and acetominophen are achiral. Malate and ibupr<strong>of</strong>in are chiral;<br />

their stereocenters are marked below with a bold dot.<br />

27


fig 2<br />

E3.7:<br />

fig 2<br />

E3.8:<br />

fig 3<br />

28<br />

<strong>Chapter</strong> 3<br />

E3.9: Because the observed optical rotation is negative, we know that there is more or the<br />

R enantiomer than the S enantiomer (if it was a 50:50 mixture, the optical rotation would<br />

be 0 o , while if it were pure (R)-carvone the optical rotation would be -61 o ).


ee = (23 x 100)/61 = 37.8%<br />

. . . so the mixture is about 69% R and 31% S.<br />

E3.10:.<br />

fig 3<br />

E3.11:<br />

There are many possible correct answers to this problem. Two are shown below.<br />

<strong>Chapter</strong> 3<br />

fig 3<br />

D-gulose is a diastereomer <strong>of</strong> D-glucose (configuration is different at two carbons). Dallose<br />

is an epimer <strong>of</strong> D-glucose (configuration is different at one carbon). Notice that the<br />

relationship between D-gulose and D-allose is that <strong>of</strong> epimers.<br />

E3.12:<br />

a) chiral<br />

b) meso (achiral)<br />

c) achiral (not meso)<br />

d) achiral (not meso)<br />

e) meso (achiral)<br />

f) chiral<br />

E3.13:<br />

29


fig 3<br />

E3.14:<br />

fig 4<br />

E3.15<br />

30<br />

<strong>Chapter</strong> 3<br />

First, we assign R/S designations to the three stereocenters in the Fischer projection <strong>of</strong> Dribose,<br />

and find that all three are R. Then, we draw the zig-zag structure, and decide<br />

whether each hydroxyl group should be drawn with a solid or dashed wedge in order to<br />

corectly show the R stereochemistry.<br />

fig 4<br />

With D-ribose accomplished, it is a simple matter to draw the zig-zag structures <strong>of</strong> the<br />

other five-carbon aldoses – just start with D-ribose, then invert the appropriate<br />

stereocenter.


fig 4<br />

E3.16<br />

<strong>Chapter</strong> 3<br />

In the figure below, homotopic hydrogens are labeled ‘h’, enantiotopic hydrogens ‘e’,<br />

and diastereotopic hydrogens ‘d’.<br />

fig 5<br />

E3.17:<br />

31


32<br />

<strong>Chapter</strong> 3<br />

fig 5<br />

The two prochiral pairs on leucine are the two hydrogens on Ca and the two methyl<br />

groups on Cb. Because there is already a true chiral center on the molecule, these pairs<br />

are both diastereotopic. Here’s how we assigned stereochemical designations to the<br />

diastereotopic methyl groups: if we pick one <strong>of</strong> the CH3 groups and give it priority #2<br />

(the H is <strong>of</strong> course priority #4), then the configuration at Cb becomes S – this methyl<br />

group is thus designated pro-S (and the other methyl group <strong>of</strong> course must be pro-R).<br />

E3.18<br />

fig 5<br />

End-<strong>of</strong>-chapter problems<br />

P3.1:


P3.2:<br />

fig 6<br />

P3.3:<br />

a) Stereocenters are marked with a bold dot.<br />

fig 6<br />

<strong>Chapter</strong> 3<br />

33


) The two fluorinated derivatives <strong>of</strong> Epivar are enantiomers.<br />

P3.4:<br />

These two structures are actually enantiomers – they are non-superimposable mirror<br />

images <strong>of</strong> each other. Notice that, even though there is no sp 3 -hybridized carbon, the<br />

overall geometry <strong>of</strong> the allene structure creates a multi-atom stereocenter.<br />

P3.5:<br />

34<br />

<strong>Chapter</strong> 3<br />

If we imagine looking down on the rings from above, we can redraw as shown below. We<br />

can also double-check the R and S designations <strong>of</strong> stereocenters to ensure that we did not<br />

draw enantiomers by mistake.<br />

a)<br />

b)<br />

c)<br />

fig 7


P3.6:<br />

a) There is only one stereocenter, and it is in the R configuration.<br />

b)<br />

fig 7<br />

P3.7:<br />

fig 7<br />

P3.8:<br />

fig 8<br />

P3.9: The two are diastereomers: two <strong>of</strong> the four stereocenters are different.<br />

<strong>Chapter</strong> 3<br />

35


P3.10:<br />

P3.11:<br />

a) enantiomers<br />

b) same compound<br />

c) constitutional isomers<br />

d) daistereomers<br />

e) same compound<br />

f) completely different compounds<br />

P3.12:<br />

a) diastereomers<br />

b) diastereomers<br />

c) epimers<br />

d) constitutional isomers<br />

All <strong>of</strong> the sugars contain three stereocenters except D-ribulose, which contains two.<br />

P3.13:<br />

a) enantiomers<br />

b) epimers<br />

c) same molecule<br />

d) completely different molecules<br />

P3.14:<br />

36<br />

<strong>Chapter</strong> 3


<strong>Chapter</strong> 3<br />

fig 8<br />

a) There are two chiral carbons in the molecule, indicated by bold dots.<br />

b) There are four pairs <strong>of</strong> prochiral hydrogens, the positions <strong>of</strong> which are indicated with<br />

arrows.<br />

P3.15:<br />

a) The molecule contains two stereocenters and has no alkene groups that can be<br />

classified E or Z; therefore, there are four possible stereoisomers.<br />

b) Positions <strong>of</strong> prochiral hydrogens are indicated with arrows.<br />

fig 8<br />

P3.16:<br />

a) The two structures are diastereomeric.<br />

b) The five-membered ring is sandwiched between the aromatic, seven-membered, and<br />

six-membered rings, with the ether oxygen as the free corner.<br />

P3.17:<br />

a) Both alkene groups are E.<br />

b) In addition to the two alkene groups, there are 10 chiral carbons in the molecule.<br />

Therefore, there are 2 12 possible stereoisomers.<br />

c) The structure shown is a diastereomer <strong>of</strong> bistramide A – the configuration <strong>of</strong> one <strong>of</strong> the<br />

alkene groups is changed from E to Z.<br />

P3.18: The two structures are exactly the same - structure B is just structure A flipped<br />

end-over-end.<br />

37


P3.19:<br />

a)<br />

38<br />

<strong>Chapter</strong> 3<br />

b) The gauche conformation makes possible the formation <strong>of</strong> an energetically-favorable<br />

intramolecular hydrogen bond (a model will help you to see this).<br />

c) When the hydroxy groups are changed to methoxy groups, intramolecular hydrogen<br />

bonding is no longer possible, and the anti conformation is expected to be lowest in<br />

energy.<br />

P3.20<br />

The highest energy conformation <strong>of</strong> 2-methylbutane is the one in which the methyl<br />

groups are eclipsed.<br />

fig 8<br />

P3.21:<br />

In the lower energy chair conformation <strong>of</strong> trans-1,2-dimethylcyclohexane, the two<br />

methyl groups are both equitorial.<br />

P3.22:<br />

In the lower energy chair conformation <strong>of</strong> cis-1-ethyl-2-methylcyclohexane, the larger<br />

ethyl group is equitorial.


P3.23:<br />

The trans diastereomer <strong>of</strong> 1,4-diethylcyclohexane, in which both ethyl groups can be<br />

equitorial, is expected to be lower in energy.<br />

fig 9<br />

P3.24:<br />

The cis diastereomer <strong>of</strong> 1,3-diethylcyclohexane, in which both ethyl groups can be<br />

equitorial, is expected to be lower in energy.<br />

fig 9<br />

P3.25:<br />

<strong>Chapter</strong> 3<br />

cis-1,2-dimethylcyclohexane is a meso compound, but the mirror plane can only be seen<br />

when the ring is in a higher-energy boat conformation (recall that, when thinking about<br />

stereochemistry, we consider all possible conformations).<br />

fig 9<br />

P3.26:<br />

39


a) A and B are enantiomers<br />

b) A and C are diastereomers<br />

c) B and C are diastereomers<br />

d) D and D are constitutional isomers<br />

P3.27:<br />

40<br />

<strong>Chapter</strong> 3


In-chapter exercises<br />

E4.1:<br />

<strong>Chapter</strong> 4<br />

<strong>Chapter</strong> 4 <strong>solutions</strong><br />

Using λυ = c, we first rearrange to υ = c/λ to solve for frequency.<br />

For light with a wavelength <strong>of</strong> 400 nm, the frequency is 7.50 x 10 14 Hz:<br />

In the same way, we calculate that light with a wavelength <strong>of</strong> 700 nm has a frequency <strong>of</strong><br />

4.29 x 10 14 Hz.<br />

To calculate corresponding energies:<br />

Using hc/λ, we find for light at 400 nm:<br />

Using the same equation, we find that light at 700 nm corresponds to 40.9 kcal/mol.<br />

E4.2:<br />

A wavenumber <strong>of</strong> 3000 cm -1 means that 3000 waves fit in one cm (0.01m):<br />

41


42<br />

<strong>Chapter</strong> 4<br />

We want to find the length <strong>of</strong> 1 wave, so we divide numerator and denominator by 3000:<br />

So 3000 cm -1 is equivalent to a wavelength <strong>of</strong> 3.33 µm.<br />

E4.3:<br />

The π → π* transition in 4-methyl-3-penten-2-one is at 236 nm, which corresponds to<br />

121 kcal/mol:<br />

E4.4:<br />

Molecule B has a longer system <strong>of</strong> conjugated π bonds, and thus will absorb at a longer<br />

wavelength. Notice that there is an sp 3 -hybridized carbon in molecule B which isolates<br />

two <strong>of</strong> the π bonds from the other three.<br />

E4.5:<br />

We use the formula:<br />

recall from your high school algebra that if y = log(x), then x = 10 y , so:<br />

. . . so the intensity <strong>of</strong> light entering the sample (I 0) is 10 times the intensity <strong>of</strong> the light<br />

(at a particular wavelength) that emerges from the sample and is detected (I). Because<br />

%T is simply the reciprocal <strong>of</strong> this ratio multiplied by 100, we find that A = 1.0<br />

corresponds to 10% transmission.<br />

E4.6: Using ε = A/c, we plug in our values for ε and A and find that c = 3.27 x 10 -5 M, or<br />

32.7 µM.<br />

E4.7:


<strong>Chapter</strong> 4<br />

Using ε = A/c and plugging in the given values for ε and A, we find that c = 16.3 ng/µL.<br />

However, this is the concentration <strong>of</strong> the diluted solution - the original solution was 20<br />

times higher (50 µL were removed and diluted to 1000 µL to take the UV measurement).<br />

Thus the concentration <strong>of</strong> the original solution is (20)(16.3) = 326 ng/µL..\<br />

E4.8:<br />

The sample is propanal. The peak at m/z = 58 is the molecular ion (parent peak), and the<br />

peak at m/z = 29 is the formyl acylium ion. The M+1 peak at m/z = 59 is a molecular ion<br />

in which one <strong>of</strong> the three carbons is a 13 C.<br />

End-<strong>of</strong>-chapter problems<br />

P4.1:<br />

Without doing any calculation, we can answer the first part <strong>of</strong> the calculation:<br />

electromagnetic waves at 3400 cm -1 are shorter than those at 1690 cm -1 (more waves fit<br />

into one centimeter) and thus correspond to a higher frequency.<br />

3400 cm -1 = 2.94 µm = 1.02 x 10 14 Hz<br />

1690 cm -1 = 5.92 µm = 5.07 x 10 13 Hz<br />

P4.2:<br />

1720 cm -1 corresponds to a wavelength <strong>of</strong> .01/1720 = 5.81 x 10 -6 m, and an energy <strong>of</strong> 4.92<br />

kcal/mol.<br />

P4.3:<br />

The triple bond in compound I is symmetric, and therefore is not IR-active, so there<br />

would be no absorbance in the carbon-carbon triple bond range (2100-2250 cm -1 ). In<br />

compound II the presence <strong>of</strong> the fluorines makes the triple bond asymmetric and IRactive,<br />

thus the alkyne peak will be observed. In compound III, we should see not only<br />

43


44<br />

<strong>Chapter</strong> 4<br />

the carbon-carbon triple bond peak but also an absorbance at approximately 3300 cm -1<br />

due to stretching <strong>of</strong> the terminal alkyne carbon-hydrogen bond.<br />

P4.4:<br />

All three spectra will have a strong carbonyl stretching peak, but the ester (compound C)<br />

carbonyl peak will be observed at a shorter wavelength compared to the ketone<br />

(compound B) and the carboxylic acid (compound A). In addition, Compound A will<br />

show a broad absorbance centered at approximately 3000 cm -1 due to carboxylic acid O-<br />

H stretching, whereas in the spectrum <strong>of</strong> compound B we should see the broad<br />

absorbance centered at approximately 3300 cm -1 from stretching <strong>of</strong> the alcohol O-H<br />

bond. Compound C will have no broad O-H stretching absorbance.<br />

P4.5:<br />

All three compounds contain alkene functional groups. However, in compound Y the<br />

alkene is symmetric and thus we would not expect to see an absorbance from C=C<br />

stretching in the 1620-1680 cm -1 range. We would expect to see this peak in the spectra <strong>of</strong><br />

compounds X and Z; in addition we would expect to see, in the compound X spectrum, a<br />

peak in the 3020 - 3080 cm -1 range due to stretching <strong>of</strong> the terminal alkene C-H bonds.<br />

P4.6:<br />

P4.7:<br />

The change in A 340 is ΔA = 0.220. Using the expression ε = A/c, we can calculate that<br />

this represents a change in the NADH concentration <strong>of</strong> 3.50 x 10 -5 M. This is in a 1 mL<br />

solution, so 3.50 x 10 -8 mol have been used up over the course <strong>of</strong> five minutes, or 7.00 x<br />

10 -9 mol (7.00 nmol) per minute on average.<br />

P4.8:<br />

Both starting compounds contain systems <strong>of</strong> conjugated π bonds which absorb in the UV<br />

range. The condensation reaction brings these two conjugated systems together to create<br />

a single, longer conjugated π system, which absorbs in the blue part <strong>of</strong> the visible<br />

spectrum.<br />

P4.9:


P4.10:<br />

<strong>Chapter</strong> 4<br />

The molecular ion and the fragments from all three <strong>of</strong> the typical ketone fragmentation<br />

patterns would all have m/z = 98.<br />

P4.11:<br />

Both molecules contain alkene and ketone functional groups, however the degree <strong>of</strong> π<br />

bond conjugation is different. Therefore, UV would be the more useful technique to<br />

distinguish the two.<br />

P4.12:<br />

Both molecules are straight-chain alkanes with a single ketone group, so their IR spectra<br />

are expected to be very similar and neither will absorb strongly in the UV range.<br />

However, the different positions <strong>of</strong> the ketone (at the C 4 vs C 5 position) will result in the<br />

formation <strong>of</strong> fragments <strong>of</strong> different masses in an MS experiment.<br />

45


In-chapter exercises<br />

E5.1:<br />

a) 8 b) 8 c) 5 d) 18<br />

E5.2:<br />

46<br />

<strong>Chapter</strong> 5<br />

<strong>Chapter</strong> 5 <strong>solutions</strong><br />

a) 1 ppm = 300 Hz, so 4.56 ppm corresponds to 1368 Hz on the 300 MHz instrument.<br />

On the 200 MHz instrument, the chemical shift is 912 Hz.<br />

b) 300 MHz instrument: 300,001,368 Hz. 200 MHz instrument: 200,000,912 Hz.<br />

E5.3: the acetone:dichloromethane ratio is 0.76:1. If it was a 1:1 ratio, the signal<br />

integration ratio would be 3:1 because acetone has 6 protons while dichloromethane has<br />

2. So you need to divide the 2.3 integral value by a factor <strong>of</strong> 3 to get 0.76.<br />

E5.4: There are three peaks: two from para-xylene and one from acetone. The acetone<br />

peak and the para-xylene methyl peak both represent six protons, so the ratio <strong>of</strong> their<br />

integration values is simply 64 to 36 or 1 to 0.56. The ratio <strong>of</strong> the para-xylene methyl<br />

peak to the para-xylene aromatic peak is 6 to 4, or 0.56 to 0.37. So the final integral ratio<br />

<strong>of</strong> acetone:methyl:aromatic signals should be 1 to 0.56 to 0.37.<br />

E5.5: The molecule contains two groups <strong>of</strong> equivalent protons: the twelve pointing to the<br />

outside <strong>of</strong> the ring, and the six pointing into the center <strong>of</strong> the ring. The molecule is<br />

aromatic, as evidenced by the chemical shift <strong>of</strong> the ‘outside’ protons’ (and the fact that<br />

there are 18 π electrons, a Hückel number). The inside protons are shielded by the<br />

induced field <strong>of</strong> the aromatic ring current, because inside the ring this field points in the<br />

opposite direction <strong>of</strong> B0. This is the source <strong>of</strong> the unusually low (negative relative to<br />

TMS) chemical shift for these protons.<br />

E5.6:


a)<br />

H b signal<br />

<strong>Chapter</strong> 5<br />

b) The figure above demonstrates that the quartet subpeaks integrate to 1:3:3:1 (eg., there<br />

are three ways for two spins to be aligned with B0 and one against B0). As an analogy, if<br />

you flip three coins at once, you have a 1 in eight chance <strong>of</strong> getting all heads, 1 in 8<br />

chance <strong>of</strong> all tails, a 3 in 8 chance <strong>of</strong> two heads and a tail, and a 3 in 8 chance <strong>of</strong> two tails<br />

and a head.<br />

E5.7:<br />

Because <strong>of</strong> the symmetry in the molecule, there are only four proton signals. Predicted<br />

splitting is indicated.<br />

E5.8:<br />

(s)<br />

H b<br />

(s)<br />

OH a<br />

O<br />

OH a<br />

H b<br />

Cl Cl<br />

Hd Hd Hc (d)<br />

Hc (d)<br />

d<br />

d<br />

H<br />

OH<br />

N<br />

NH 3<br />

(recall that splitting is generally not seen with protons on heteroatoms)<br />

E5.9:<br />

t<br />

t<br />

47


48<br />

H b<br />

5.64 ppm<br />

1692 Hz<br />

J ab = 10.5 Hz<br />

J bc = 1.5 Hz<br />

chemical shifts <strong>of</strong> subpeaks (Hz)<br />

1698.0<br />

1696.5<br />

1687.5<br />

1686.0<br />

E5.10: We can think <strong>of</strong> this signal as being a triplet <strong>of</strong> triplets, but because the two<br />

coupling constants are very close, what we would actually see is a 1,2,3,2,1 pentet.<br />

E5.11:<br />

H b<br />

J ab = J bc<br />

a) 8 signals (each carbon is different)<br />

b) 11 signals (the two enantiotopic CH2CH3 groups are NMR-equivalent)<br />

c) 6 signals (each carbon is different)<br />

d) 16 signals (the fluorobenzene group only contributes 4 signals due to symmetry)<br />

E5.12:<br />

a) 5 signals<br />

b) 5 signals<br />

c) 3 signals<br />

3<br />

3<br />

1 2 2 1<br />

3 3<br />

<strong>Chapter</strong> 5


d) 6 signals<br />

E5.13:<br />

E5.14:<br />

E5.15:<br />

29.29<br />

O<br />

H 3C C C<br />

O<br />

O<br />

pyruvate: spectrum b<br />

18.95<br />

H 3N<br />

53.25<br />

CH 3<br />

O<br />

207.85<br />

O<br />

alanine (spectrum c)<br />

Cl<br />

CH 3<br />

125.4<br />

172.69<br />

H 3C CH 3<br />

compound A<br />

178.54<br />

O<br />

H 2C C<br />

136.5<br />

C OCH3<br />

CH 3<br />

18.4<br />

compound B<br />

167.9<br />

O<br />

51.8<br />

182.63 45.35 73.06<br />

O<br />

OH<br />

O<br />

O<br />

(S)-malate: spectrum d<br />

168.10<br />

Cl<br />

O<br />

N<br />

NH 2<br />

N<br />

H<br />

159.91<br />

cytosine (spectrum a)<br />

Cl<br />

compound C<br />

183.81<br />

95.79<br />

144.05<br />

<strong>Chapter</strong> 5<br />

E5.16: This benzylic carbocation is especially stable, as the positive charge is stabilized<br />

by two benzene rings.<br />

49


End-<strong>of</strong>-chapter problems<br />

50<br />

<strong>Chapter</strong> 5<br />

P5.1: (structures n and o are chiral, and thus we must consider diastereotopic protons)<br />

structure # 1 H peaks # 13 C peaks<br />

a 4 5<br />

b 2 3<br />

c 4 4<br />

d 5 4<br />

e 2 3<br />

f 3 4<br />

g 4 6<br />

h 3 6<br />

i 1 2<br />

j 2 2<br />

k 3 4<br />

l 5 5<br />

m 4 3<br />

n 6 4<br />

o 10 5<br />

P5.2: (valine and leucine both have diastereotopic (and thus nonequivalent) methyl<br />

groups.<br />

Amino acid # 13 C signals<br />

glycine 2<br />

alanine 3<br />

valine 5<br />

leucine 6<br />

isoleucine 6<br />

phenylalanine 7<br />

tyrosine 7<br />

tryptophan 11<br />

methionine 5<br />

cysteine 3<br />

serine 3<br />

threonine 4<br />

arginine 6<br />

lysine 6<br />

histidine 6<br />

proline 5<br />

glutamate 5<br />

aspartate 4<br />

glutamine 5<br />

asparagine 4


P5.3:<br />

<strong>Chapter</strong> 5<br />

a) On a 300 MHz instrument, 1 ppm = 300 Hz, thus a chemical shift <strong>of</strong> 2.9634 ppm<br />

corresponds to 889.0 Hz. The downfield subpeak is 889.0 Hz plus ½ <strong>of</strong> the coupling<br />

constant, or 893.1 Hz. Correspondingly, the upfield subpeak is at 889.0 Hz minus ½ <strong>of</strong><br />

the coupling constant, or 884.9 Hz.<br />

b) In a 100 MHz instrument, 1 ppm = 100 Hz, so the chemical shift <strong>of</strong> 2.9634 ppm<br />

corresponds to 296.34 Hz. Adding and subtracting ½ <strong>of</strong> the coupling constant gives us<br />

chemical shift values for the two subpeaks <strong>of</strong> 300.4 Hz and 292.2 Hz.<br />

P5.4: The chemical shift, in Hz, <strong>of</strong> the overall signal is (1.7562 ppm)(300 Hz/ppm) =<br />

526.9 Hz. The furthest downfield subpeak has a chemical shift <strong>of</strong> 538.3 Hz (see the figure<br />

below). The resonance frequency is this number plus 300 million Hz, or 300,000,538.3<br />

Hz.<br />

P5.5:<br />

538.3 Hz<br />

7.6 Hz 3.8 Hz<br />

A pair <strong>of</strong> doublets indicates para disubstitution:<br />

H a<br />

H b<br />

1.7562 ppm<br />

(526.9 Hz)<br />

X<br />

Y<br />

H a<br />

Hb<br />

51


52<br />

<strong>Chapter</strong> 5<br />

Note: In problems 6-13, references are given to the structure in the Aldrich Library <strong>of</strong><br />

NMR Spectra (volume - page)<br />

P5.6:<br />

Spectrum 1: 1-941C (structure D)<br />

Spectrum 2: 1-941B (structure F)<br />

Spectrum 3: 1-940B (structure C)<br />

Spectrum 4: 1-939C (structure B)<br />

Spectrum 5: 1-938C (structure A)<br />

Spectrum 6: 1-940A (structure E)<br />

P5.7:<br />

Spectrum 7: 1-735B, (structure K)<br />

Spectrum 8: 1-735A ( structure J)<br />

Spectrum 9: 1-733C (structure I)<br />

Spectrum 10: 1-729B (structure G)<br />

Spectrum 11: 1-729A (structure H)<br />

Spectrum 12: 1-895C (structure L)<br />

P5.8:<br />

Spectrum 13: 1-723B (structure M)<br />

Spectrum 14: 1-723C (structure O)<br />

Spectrum 15: (1-724A, structure P)<br />

Spectrum 16: (1-724C, structure N)<br />

Spectrum 17: (1-724B, structure R)<br />

Spectrum 18: (1-722-C, structure Q)


P5.9:<br />

P5.10:<br />

2.12<br />

H3C 7.15-7.29<br />

7.10<br />

O<br />

Spectrum 23: 2-794B<br />

V<br />

(spectrum 22: 2-789A<br />

6.86 3.61 3.78<br />

O<br />

O<br />

Cl<br />

S T U<br />

2.73<br />

2.86<br />

2.12<br />

Spectrum 25: (2-939A, structure FF)<br />

Spectrum 26: (2-943C, structure BB)<br />

Spectrum 27: (2-790A, structure AA)<br />

Spectrum 28: (2-1201A, structure CC)<br />

Spectrum 29: (2-455B, structure EE)<br />

Spectrum 30: (3-85C, structure DD)<br />

P5.11:<br />

Spectrum 31: (2-265C, structure HH)<br />

Spectrum 32: (2-266A, structure KK)<br />

Spectrum 33: (2-1212B, structure LL)<br />

Spectrum 34: (2-1366A, structure GG)<br />

Spectrum 35: (2-1366B, structure JJ)<br />

Spectrum 36: (2-1368C, structure II)<br />

O<br />

Spectrum 21: 2-940B<br />

O<br />

7.50<br />

8.38<br />

7.75<br />

W<br />

7.81<br />

O<br />

8.17<br />

Spectrum 20: 2-938B<br />

9.98<br />

10.14<br />

7.23-7.30<br />

Spectrum 24: 2-791C<br />

7.31<br />

2.43<br />

H3C 7.77<br />

X<br />

3.53<br />

<strong>Chapter</strong> 5<br />

O<br />

O<br />

9.94<br />

Spectrum 19: 2-938C<br />

53


P5.12: References to the Aldrich catalog <strong>of</strong> 1 H-NMR spectra are provided for each<br />

structure)<br />

a ) 1-734A<br />

54<br />

O<br />

b) Molecular formula: C7H14O2<br />

1-905B<br />

O<br />

O<br />

c) Molecular formula: C5H12O<br />

1-170B<br />

OH<br />

d) Molecular formula: C10H12O<br />

2-789B<br />

O<br />

P5.13: 13 C-NMR data is given for the molecules shown below. Complete the peak<br />

assignment column <strong>of</strong> each NMR data table.<br />

a) (1-895C)<br />

1<br />

O<br />

O<br />

2<br />

3<br />

4<br />

<strong>Chapter</strong> 5


δ (ppm) carbon #(s)<br />

161.12 1<br />

65.54 2<br />

21.98 3<br />

10.31 4<br />

b) (1-735A)<br />

1<br />

2 3<br />

O<br />

5<br />

4<br />

6<br />

δ (ppm) carbon #(s)<br />

194.72 4<br />

149.10 3<br />

146.33 2<br />

16.93 5<br />

14.47 1<br />

12.93 6<br />

c) (1-938)<br />

1<br />

2<br />

O O<br />

O 3 4 5 O<br />

8 9<br />

10 11<br />

δ (ppm) carbon #(s)<br />

171.76 3, 5<br />

60.87 2, 6<br />

58.36 4<br />

24.66 8, 9<br />

14.14 1, 7<br />

8.35 10, 11<br />

6<br />

7<br />

<strong>Chapter</strong> 5<br />

55


d) (2-1212B)<br />

1<br />

H3C 56<br />

O<br />

O<br />

2<br />

δ (ppm) carbon #(s)<br />

173.45 2<br />

155.01 7<br />

130.34 6, 8<br />

125.34 4<br />

115.56 5, 9<br />

52.27 1<br />

40.27 3<br />

e) (2-455B)<br />

3<br />

4<br />

2<br />

5<br />

1<br />

8<br />

6<br />

N<br />

3<br />

10<br />

δ (ppm) carbon #(s)<br />

147.79 1<br />

129.18 2, 6<br />

115.36 3, 5<br />

111.89 4<br />

44.29 7, 8<br />

12.57 9, 10<br />

5<br />

7<br />

4<br />

9<br />

6<br />

9<br />

OH<br />

7<br />

8<br />

P5.14: The structure is 3-methyl-2-butanone<br />

<strong>Chapter</strong> 5<br />

P5.15: First, assign the peaks. 1-bromopropane: Ha is the triplet at 3.4 ppm, Hb is the<br />

sextet at 1.9 ppm, Hc is the triplet at 1.0 ppm. 2-bromopropane: Hd is the doublet at 1.7<br />

ppm, He is the septet at 4.3 ppm. Now, just add up the integrations for each molecule


<strong>Chapter</strong> 5<br />

and figure the percentage. 1-bromopropane: 0.661 + 0.665 + 1.00 = 2.326. 2bromopropane:<br />

0.0735 + 0.441 = 0.5145. Percent 2-bromopropane is (0.5145)*100 /<br />

(0.5145 + 2.326) = 18.1%.<br />

Notice that even if some peaks overlapped, you could still get this number as long as you<br />

could integrate one signal on each molecule: you could, for example, do the same<br />

calculation (and get the same result) by comparing integrations <strong>of</strong> Hc and Hd signals, and<br />

just scale based on the fact that the Hc signal represents three protons while the Hd signal<br />

represents six protons.<br />

H a<br />

H a<br />

H b H b<br />

C C C<br />

Br<br />

H c<br />

integration:<br />

Challenge problems<br />

H c<br />

H c<br />

H c: 1.0 ppm (t)<br />

1.00<br />

3 protons<br />

percent 2-bromopropane =<br />

= 0.33<br />

0.0735<br />

0.0735 + 0.33<br />

H d<br />

H d<br />

H e Br<br />

C C C<br />

H d<br />

integration:<br />

H d<br />

H d<br />

x 100 = 18.2 %<br />

H d<br />

H d: 1.7 ppm (d)<br />

0.441<br />

6 protons<br />

= 0.0735<br />

C5.1: The 13 C-NMR signal for the CDCl3 solvent appears as a 1:1:1 triplet due to 1-bond<br />

coupling with the deuterium. Protons and 13 C nuclei both have a spin quantum number I<br />

= ½, which means that they can assume two spin energy levels (+ ½ and – ½ ) when<br />

placed in a strong external magnetic field. Deuterium, however, has a spin number I = 1,<br />

meaning that it can assume three spin energy level (-1, 0, +1) in the external field. So<br />

while a proton splits the signal from a neighboring nucleus into a doublet, a deuterium<br />

splits its neighbor’s signal into three sub-peaks <strong>of</strong> equal height.<br />

C5.2: See J. Chem. Educ. 2000, 77, 363.<br />

C5.3: First, convert the chemical shift data from ppm to Hz, plot the data, then simply<br />

compute the distance between the appropriate sub-peaks. For example, the distance<br />

between the two subpeaks in the Ha doublet (9.67 ppm) is about 7.8 Hz, which is the<br />

value <strong>of</strong> Jab. To get Jbc, it would make sense to look at the Hc doublet – except that this<br />

signal overlaps with the aromatic signals. Alternatively, we can look at the Hb signal,<br />

which is centered at 6.69 ppm. This is a doublet <strong>of</strong> doublets, but as we already know Jab<br />

we can measure Jbc (approximately 16 Hz).<br />

57


58<br />

H b<br />

J bc = 16 Hz<br />

J ab = 7.8 Hz<br />

<strong>Chapter</strong> 5


In-chapter exercises<br />

E6.1:<br />

fig 1<br />

E6.2:<br />

E6.3<br />

<strong>Chapter</strong> 6<br />

<strong>Chapter</strong> 6 <strong>solutions</strong><br />

59


fig 1<br />

End-<strong>of</strong>-chapter problems<br />

P6.1:<br />

Left side: the nucleophile is an amine, the electrophile is the methyl carbon, and the<br />

leaving group is a sulfide.<br />

60<br />

<strong>Chapter</strong> 6<br />

Right side: The nucleophile is a thiolate ion, the electrophile is the carbon atom <strong>of</strong> an<br />

alkyl diphosphate, and diphosphate is the leaving group.<br />

P6.2:<br />

a)<br />

fig 2<br />

b) In step 1, the nucleophile is the hydroxide oxygen, and the electrophile is the carbonyl<br />

carbon <strong>of</strong> the thioester.<br />

P6.3:<br />

a)


)<br />

c)<br />

d)<br />

fig 2<br />

<strong>Chapter</strong> 6<br />

61


e)<br />

fig 3<br />

f) Bold dots show the two carbons that form a new bond in this reaction step.<br />

P6.4:<br />

The C to D step has the highest activation energy, and thus is the slowest, ratedetermining<br />

step.fig 3<br />

P6.5:<br />

62<br />

<strong>Chapter</strong> 6


In-chapter exercises<br />

E7.1:<br />

fig 1<br />

E7.2:<br />

a) pKa ~ 12 (ammonium)<br />

b) pKa ~ 36 (amine)<br />

c) pKa ~ 10 (thiol)<br />

d) pKa ~ 4.5 (carboxylic acid)<br />

e) pKa ~ 4.5 (carboxylic acid)<br />

Conjugate bases are shown below:<br />

<strong>Chapter</strong> 7<br />

<strong>Chapter</strong> 7 <strong>solutions</strong><br />

63


fig 1<br />

E7.3:<br />

a)<br />

K eq ~ 10 (10.5 - 17) = 10 -6.5 = 7.1 x 10 -5<br />

b)<br />

K eq = 10 (4.8 - 19) = 10 -14.2 = 6.3 x 10 -15<br />

c)<br />

K eq ~ 10 (18-10) = 10 8<br />

d)<br />

64<br />

<strong>Chapter</strong> 7


<strong>Chapter</strong> 7<br />

K eq ~ 10 (15.7-10) = 10 5.7 = 5.0 x 10 5<br />

fig 1<br />

E7.4:<br />

Problem: If an arginine side chain in a protein is exposed to buffer with pH = 8.5,<br />

to what extent (expressed as a percentage) is it deprotonated?<br />

If we approximate the pK a <strong>of</strong> an arginine side chain as 12.5, and use the Henderson -<br />

Hasselbalch equation:<br />

. . . so the arginine side chain is essentially 100% protonated in this buffer.<br />

E7.5:<br />

Using the Henderson-Hasselbalch equation, we find that the ratio [HA]/[A-] = 10 (5.2-4.8) =<br />

10 0.4 = 2.5. If we express this ratio as 5:2, then we can see that for every 7 molecules <strong>of</strong><br />

acetic acid in this buffer, 5 are protonated (uncharged) and 2 are deprotonated (negatively<br />

charged). From this we can easily calculate that about 29% <strong>of</strong> acetic acid molecules are<br />

negatively charged.<br />

E7.6:<br />

The thiol is the most acidic group, and thus is the group that loses the proton.<br />

65


fig 2<br />

E7.7:<br />

66<br />

<strong>Chapter</strong> 7<br />

C is an amine, and thus is less acidic than A and B, which are (protonated) ammonium<br />

ions. A is more acidic than B, because the electron lone pair on the conjugate base <strong>of</strong> A<br />

is stabilized by resonance.<br />

fig 2<br />

E7.8:<br />

The acidity ranking is A>C>B. In acid A, the inductive effect stabilizing the negative<br />

charge on the conjugate base is strongest, because chlorine is more electronegative than<br />

bromine. Acid C is stronger than B because in C, the electron-withdrawing bromine is<br />

closer by one bond to the negative charge on the conjugate base.<br />

E7.9:<br />

Carbons on which the negative charge can be localized are indicated with a bold dot.<br />

fig 2<br />

E7.10:<br />

The negative charge on the conjugate base <strong>of</strong> picric acid can be delocalized to three<br />

different nitro oxygen atoms (in addition to the phenolate oxygen).


fig 2<br />

E7.11:<br />

<strong>Chapter</strong> 7<br />

A is a weaker base than B: the lone pair on the amine nitrogen <strong>of</strong> A can be delocalized to<br />

the aldehyde oxygen, whereas this is not possible with B.<br />

Fig 2<br />

E7.12:<br />

a) The α-protons are circled:<br />

fig 3<br />

b)<br />

67


fig 3<br />

E7.13:<br />

68<br />

<strong>Chapter</strong> 7<br />

fig 3<br />

The non-bridging oxygens on the β (outer) phosphate are most basic, because protonation<br />

here results in there being a greater distance between the two remaining negative charges.<br />

E7.14:<br />

In this situation, the lysine side chain is in a hydrophobic environment. Therefore, the<br />

protonated (positively charged ammonium) state is highly destabilized, and the<br />

ammonium group is more acidic (because by giving up a proton it will rid itself <strong>of</strong> its<br />

positive charge). The overall effect <strong>of</strong> the protein microenvironment is pK a lowering in<br />

this case.<br />

E7.15:<br />

The metal ion acts as a Lewis acid, accepting electron density from the negativelycharged<br />

oxygen <strong>of</strong> the enolate that forms upon deprotonation. This, <strong>of</strong> course, is a


<strong>Chapter</strong> 7<br />

stabilizing effect, and thus the proximity <strong>of</strong> the metal ion serves to decrease the pK a <strong>of</strong> the<br />

α-protons.<br />

fig 4<br />

End-<strong>of</strong>-chapter problems<br />

P7.1:<br />

The interior Asp has a higher pK a (is less acidic). It is in a hydrophobic environment,<br />

meaning that that the charged (aspartate) form <strong>of</strong> the side chain will be highly<br />

destabilized relative to the uncharged aspartic acid form.<br />

P7.2:<br />

In all cases the pK a <strong>of</strong> the amino acid side chain (or <strong>of</strong> water, for part e) is expected to be<br />

lower due to the proximity <strong>of</strong> the cationic magnesium ion. The positive charge on the<br />

metal ion is expected to stabilize the negatively-charged conjugate base form <strong>of</strong> Glu, Tyr,<br />

and water, and to destabilize the positively charged, conjugate acid forms <strong>of</strong> Lys and His.<br />

P7.3:<br />

a) The base on the right is stronger<br />

b) The base on the left is stronger<br />

c) The base on the left is stronger<br />

d) The base on the right is stronger<br />

e) The base on the left is stronger<br />

f) The base on the right is stronger<br />

g) The base on the right is stronger<br />

h) The base on the right is stronger<br />

i) The base on the left is stronger<br />

j) The base on the left is stronger<br />

69


70<br />

<strong>Chapter</strong> 7<br />

P7.4: The positive charge on the protonated form <strong>of</strong> arginine can be delocalized by<br />

resonance to all three <strong>of</strong> the nitrogens – this stabilizes the conjugate acid form (ie. makes<br />

it a weaker acid). On the protonated form <strong>of</strong> lysine, by contrast, the positive charge is<br />

‘stuck’ on the single nitrogen (see the structure <strong>of</strong> lysine in chapter 6). Because the<br />

positive charge <strong>of</strong> <strong>of</strong> lysine is not stabilized by resonance, lysine is more likely to give up<br />

a proton and lose the charge.<br />

fig 2<br />

P7.5: The ester oxygen acts as an electron-donating group by resonance. This electrondonating<br />

property destabilizes the negative charge on the enolate form, making the αproton<br />

less acidic. This argument also holds true for thioesters.<br />

fig 3<br />

P7.6:<br />

a) The most acidic proton on tetracycline is indicated below. Notice that the negative<br />

charge on the conjugate base can be delocalized to two carbonyl oxygens.


<strong>Chapter</strong> 7<br />

fig 4<br />

When the second most acidic proton is abstracted by a base, the resulting negative charge<br />

can be delocalized to one carbonyl oxygen (in addition to three aromatic carbons). The<br />

2 nd conjugate base <strong>of</strong> tetracycline is shown below (having fully reacted with two molar<br />

equivalents <strong>of</strong> strong base).<br />

fig 4<br />

P7.7:<br />

71


fig 5<br />

P7.8:<br />

fig 5<br />

P7.9:<br />

72<br />

<strong>Chapter</strong> 7<br />

One nitrogen is simply a primary amine, and as such is basic. The other nitrogen is<br />

‘pyrrole-like’, meaning that its lone pair is part <strong>of</strong> an aromatic sextet, and is not available<br />

for bonding to another proton.


fig 5<br />

P7.10:<br />

The alkyl amine nitrogen is most basic, the ‘pyrrole-like’ nitrogen is least basic.<br />

fig 5<br />

P7.11:<br />

<strong>Chapter</strong> 7<br />

Proton H a is more acidic. The negative charge on the conjugate base that results from the<br />

abstraction <strong>of</strong> H a can be delocalized to both oxygens plus a carbon, whereas the negative<br />

charge from abstraction <strong>of</strong> H b can be delocalized just to the two oxygens.<br />

fig 5<br />

P7.12:<br />

73


74<br />

<strong>Chapter</strong> 7<br />

Of the four protons, H a is the least acidic. The negative charge that results from<br />

abstraction <strong>of</strong> H a can be delocalized to only one oxygen atom, whereas the charge<br />

resulting from abstraction <strong>of</strong> either H b, H c, and H d can be delocalized to two oxygens in<br />

each case.<br />

fig 6<br />

P7.13:<br />

The total charge will be very close to –3.0. At pH = 7.3, the N-terminus proline (pK a =<br />

10.6) will be fully protonated, and will contribute a charge <strong>of</strong> +1. This is balanced,<br />

however, by a negative charge on the terminal glutamate (pKa = 2.2). Three <strong>of</strong> the seven<br />

amino acids in the peptide have ionizable side chains: an aspartate (D) and two<br />

glutamates (E). The pK a values for these side chains are 3.7 and 4.3, respectively, so at<br />

pH 7 all three will be fully ionized, leading to a total peptide charge <strong>of</strong> –3.<br />

P7.14:<br />

There are three ionizable groups on this peptide: the terminal amino group on Asp (pK a ~<br />

9.6), the side-chain carboxylate group on Asp (pK a ~ 3.7) and the terminal carboxylate on<br />

Ile (pK a ~ 2.4). For each buffer, we can use the Henderson-Hasselbalch equation to<br />

determine the charged / uncharged ratio for each group.<br />

a) At pH = 4.0:<br />

Asp (terminal amino) [HA + ] / [A] = 10 (9.6-4.0) = 10 5.6 = 4.0 x 10 5 . At this pH the terminal<br />

amino group is essentially 100% protonated and positively charged, so this group<br />

contributes a charge <strong>of</strong> +1.<br />

Asp (side chain): [HA] / [A - ] = 10 (3.7 - 4.0) = 10 (-0.3) = 0.50. Approximately 2 out <strong>of</strong> every 3<br />

side chains is deprotonated and negatively charged, so overall this group contributes a<br />

charge <strong>of</strong> -0.67.<br />

Ile (terminal carboxylate): ): [HA] / [A - ] = 10 (2.4 - 4.0) = 10 (-1.6) = 0.025. Most, but not all <strong>of</strong><br />

the terminal carboxylates are deprotonated and negatively charged. We can calculate the<br />

percentage that are protonated:


<strong>Chapter</strong> 7<br />

. . .thus about 97.6% are deprotonated and negatively charged. This group contributes an<br />

overall charge <strong>of</strong> -0.98.<br />

In a buffer <strong>of</strong> pH 4.0, the total charge on the dipeptide will be:<br />

(+1) + (-0.67) + (-0.98) = -0.65.<br />

b) in a buffer with pH = 7.3, the total charge on the dipeptide will be close to -2 (the<br />

terminal amino group is 100% protonated, both carboxylate groups are 100%<br />

deprotonated)<br />

c) in a buffer with pH = 9.6, the total charge on the dipeptide will be close to -2.5 (in this<br />

basic buffer, the terminal amino group is 50% deprotonated, and so only contributes a<br />

charge <strong>of</strong> +0.5).<br />

P7.15:<br />

K eq = 10 -8.2 = 6.3 x 10 -9<br />

75


76<br />

<strong>Chapter</strong> 7<br />

fig 6<br />

How did we pick the most basic group on the Y species? We have four choices: a<br />

primary amine, a 'pyrrole-like' amine, and two carboxylates. We know that pyrrole-like<br />

amines are not basic, and we can look at our pK a table to remind ourselves that primary<br />

amines are more basic than carboxylates.<br />

fig 7<br />

K eq = 10 (10-19) = 10 -9<br />

P7.16:


fig 6<br />

<strong>Chapter</strong> 7<br />

P7.17: Aqueous <strong>solutions</strong> <strong>of</strong> 'Tris' and 'HEPES' are very commonly used as buffers in<br />

biochemistry and molecular biology laboratories. You make two buffer <strong>solutions</strong>: One is<br />

50 mM Tris at pH 7.0, the other 50 mM HEPES at pH 7.0. For each solution, calculate<br />

the concentration <strong>of</strong> buffer molecules that are in their charged (ionic) protonation states.<br />

Tris: Using the Henderson-Hasselbalch equation, we find that the ratio [HA + ] / [A] at this<br />

pH is 10 (8.1-7.0) = 10 1.1 = 12.6. The percentage <strong>of</strong> HA + is thus (12.6/13.6)*100 = 93%. The<br />

concentration <strong>of</strong> protonated (positively charged) Tris is (0.93)(50 mM) = 46.5 mM.<br />

HEPES: the most acid proton is on the sulfate group (the negative charge on the<br />

conjugate base can be delocalized to two oxygens).<br />

[HA] / [A - ] = 10 0.5 = 3.2. This means 76% is protonated (uncharged) at this pH, and 24%<br />

is deprotonated (negatively charged). Thus the concentration <strong>of</strong> negatively-charged<br />

HEPES is (0.24)(50mM) = 12 mM.<br />

77


In-chapter exercises<br />

E8.1:<br />

fig 1<br />

E8.2:<br />

78<br />

<strong>Chapter</strong> 8<br />

<strong>Chapter</strong> 8 <strong>solutions</strong><br />

Notice that the reaction occurs with inversion <strong>of</strong> configuration: the leaving group (I) was<br />

pointing out <strong>of</strong> the plane <strong>of</strong> the page, while the nucleophile (CH 3S - ) attacks from behind,<br />

and ends up pointing into the plane <strong>of</strong> the page.


fig 1<br />

E8.3:<br />

The intermediate species is a carbocation that forms after the bromine leaves:<br />

E8.4:<br />

fig 1<br />

E8.5:<br />

<strong>Chapter</strong> 8<br />

Reaction A is a concerted (one-step) reaction, essentially a collision between two species.<br />

Therefore, if the concentration <strong>of</strong> nucleophile (CH 3S - ) were doubled in the solution, the<br />

rate <strong>of</strong> the reaction should also double. You should recall learning about rate expressions<br />

in your general chemistry course - this is an example <strong>of</strong> a second order rate expression,<br />

where the rate depends on some rate constant (k) and the product <strong>of</strong> two concentration:<br />

rate = k[CH 3I] [CH 3S - ]<br />

(If this is unfamiliar to you, now would be a good time for a quick review <strong>of</strong> rate<br />

expressions in your general chemistry textbook!)<br />

Reaction B is a two-step reaction. The first step – the breaking <strong>of</strong> the C-Br bond – is the<br />

slow, rate determining step, and does not involve the CH 3SH nucleophile at all.<br />

Therefore, changing the concentration <strong>of</strong> the nucleophile should have no effect on the<br />

rate <strong>of</strong> the reaction. (If the concentration <strong>of</strong> CH 3SH were doubled, the second step would<br />

occur twice as fast – but the rate <strong>of</strong> the first, slower step would not change, and so the<br />

overall rate <strong>of</strong> the reaction would not be affected). The rate expression in this case is first<br />

order (it depends on the concentration <strong>of</strong> only one reactant):<br />

rate = k[C(CH 3) 3Br]<br />

79


E8.6:<br />

80<br />

<strong>Chapter</strong> 8<br />

The side chain on serine – a primary alcohol – is in general a better nucleophile then the<br />

side chain on tyrosine, which is a phenol. Recall (section 7.4A) the argument for why<br />

phenolate (deprotonated phenol) is less basic than ethanoate (deprotonated ethanol) –<br />

some <strong>of</strong> the electron density <strong>of</strong> the oxygen atom in phenolate is delocalized to the<br />

aromatic ring. The same argument holds true when comparing the nucleophilicity <strong>of</strong><br />

serine to tyrosine: electron density on the tyrosine oxygen is stabilized somewhat by<br />

resonance with the benzene ring, and is therefore less reactive. Steric factors also come<br />

into play - the large phenyl ring makes nucleophilic attack more difficult for the tyrosine.<br />

E8.7:<br />

A cysteine, with its thiol side chain, is very nucleophilic - much more so than a<br />

methionine with its sulfide group. The thiol sulfur atom is less hindered than the sulfur<br />

atom on a sulfide, and in addition the thiol proton can be removed by a base as the<br />

nucleophilic attack takes place. The methyl group on a methionine side chain, in<br />

contrast, cannot be removed by a base, so that the product <strong>of</strong> a nucleophilic attack by<br />

methionine will contain a positive formal charge on the sulfur.<br />

fig 2<br />

E8.8:<br />

a) phenolate is the stronger base (compare the pK as <strong>of</strong> the conjugate acids), and is also the<br />

stronger nucleophile – the electrons on the nucleophilic oxygen atoms are more reactive.<br />

b) water is nucleophilic, hydronium ion is not – the difference is in the protonation state<br />

(deprotonation increases nucleophilicity)<br />

c) trimethyl amine is less hindered, and therefore is the better nucleophile.<br />

d) iodide ion is more polarizable, and therefore is a better nucleophile (even though it is<br />

the weaker base).<br />

e) CH 3NH - is anionic and deprotonated – it is the stronger nucleophile.<br />

E8.9:


<strong>Chapter</strong> 8<br />

In the carbocation on the left, the positive charge is located in a position relative to the<br />

nitrogen such that the lone pair <strong>of</strong> electrons on the nitrogen can be donated to fill the<br />

empty orbital. This is not possible for the carbocation species on the right.<br />

fig 2<br />

E8.10:<br />

fig 2<br />

E8.11:<br />

a) 1 (tertiary vs. secondary carbocation)<br />

b) equal<br />

c) 1 (tertiary vs. secondary carbocation)<br />

d) 2 (positive charge is further from electron-withdrawing fluorine)<br />

e) 1 (lone pair on nitrogen can donate electrons by resonance)<br />

f) 1 (allylic carbocation – positive charge can be delocalized to a second carbon)<br />

E8.12:<br />

81


82<br />

<strong>Chapter</strong> 8<br />

a) B (thiolate is a weaker base/better leaving group than alcohol)<br />

b) A (sulfide is better leaving group than thiolate (sulfide is neutral and not at all basic)<br />

c) A (bromide ion is a weaker base/better leaving group than acetate – compare pK a<br />

values <strong>of</strong> HBr and acetic acid).<br />

d) B (inductive electron-withdrawing effect <strong>of</strong> fluorines make trifluoroacetate a weaker<br />

base/better leaving group).<br />

E8.13:<br />

E8.14:<br />

a) Notice that the oxygen nucleophile attacks from behind the plane <strong>of</strong> the page, while the<br />

chlorine is pointing out <strong>of</strong> the plane <strong>of</strong> the page – this is a backside attack by the<br />

nucleophile.<br />

fig 3


<strong>Chapter</strong> 8<br />

b) Because both the nucleophilic oxygen and the chlorine leaving group are oriented on<br />

the same side <strong>of</strong> the ring, backside attack by the nucleophile is impossible. In order for a<br />

displacement to occur, it would have to be a stepwise (S N1) mechanism. The observation<br />

that compound A forms an epoxide under these conditions but compound B does not<br />

strongly suggests that the reaction proceeds by an S N2 mechanism, with a requirement for<br />

backside attack by the nucleophile.<br />

E8.15:<br />

fig 3<br />

End-<strong>of</strong>-chapter problems<br />

P8.1:<br />

All <strong>of</strong> the molecules in question are primary alkyl bromides, and the nucleophile is a very<br />

powerful thiolate ion – we are talking here about SN 2 reactions. The rate <strong>of</strong> substitution<br />

depends on the amount <strong>of</strong> steric hindrance in the electrophile – the less hindrance, the<br />

faster the substitution reaction. The order is:<br />

fastest D > B > A > C > slowest<br />

P8.2:<br />

fig 4<br />

P8.3:<br />

83


84<br />

<strong>Chapter</strong> 8<br />

Carbocation A is more stable: it has a longer system <strong>of</strong> conjugated π bonds, and as a<br />

consequence the charge can be delocalized over five different carbons (as opposed to four<br />

carbons for B).<br />

P8.4:<br />

Here we are looking at periodic trends and steric hindrance. Nucleophilicity increases<br />

going down a column <strong>of</strong> the periodic table, so A and C, with phosphorus atoms, are<br />

expected to be more nucleophilic than B and D. C is more nucleophilic than A, because<br />

the three methyl groups on C are the cause <strong>of</strong> less steric hindrance than the three ethyl<br />

groups on A. Using the same reasoning, we can see that B should be more nucleophilic<br />

than D, because <strong>of</strong> the bulky phenyl group on D. The trend is:<br />

most nucleophilic C > A > B > D least nucleophilic<br />

P8.5:<br />

fig 4<br />

P8.6:<br />

The reaction shown in part e) is intramolecular (one molecule reacting with itself), and<br />

thus will likely have a first order rate expression.<br />

P8.7:<br />

a) water or hydroxide ion<br />

b) CH 3S - or CH 3OH<br />

c) CH 2S - or CH 3SH<br />

d) acetate ion or hydroxide ion<br />

e) diethyl sulfide or diethyl ether<br />

f) dimethylamine or diethylether


g) trimethylamine or 2,2-dimethylpropane<br />

P8.8:<br />

<strong>Chapter</strong> 8<br />

The major product will be dimethyl sulfide (CH 3SCH 3), because CH 3S - is a better<br />

nucleophile than CH 3O - and will react faster with methyl bromide in an S N2 displacement.<br />

P8.9:<br />

a) the compound on the left<br />

b) the compound on the right<br />

c) the compound on the right<br />

d) the compound on the left<br />

e) the compound on the left<br />

f) the compound on the left<br />

g) the compound on the right<br />

P8.10:<br />

fig 4<br />

P8.11:<br />

85


fig 5<br />

P8.12:<br />

fig 5<br />

P8.13:<br />

86<br />

<strong>Chapter</strong> 8


fig 6<br />

P8.14:<br />

<strong>Chapter</strong> 8<br />

The first step in an S N1 reaction is carbocation formation. However, the rigid ‘bicyclo’<br />

structure <strong>of</strong> the starting material prevents a hypothetical carbocation intermediate from<br />

adopting trigonal planar geometry (make a model to see this better). Consequently, there<br />

is a large energy barrier for carbocation formation.<br />

fig 6<br />

Challenge problems<br />

C8.11:<br />

See the following references for a solution:<br />

March, Jerry, Advanced Organic Chemistry: Reactions and Mechanisms, and Structure,<br />

4th ed. (1992), p. 296. (Use the 'author index' to find a discussion <strong>of</strong> this paper in more<br />

recent editions <strong>of</strong> the book).<br />

J. Chem Soc. 1935, 1525. (This is the original paper, but may be hard to locate.)<br />

87


In-chapter exercises<br />

E9.1:<br />

88<br />

<strong>Chapter</strong> 9<br />

<strong>Chapter</strong> 9 <strong>solutions</strong><br />

The electrophilic carbon is primary, and a carbocation intermediate would not be<br />

stabilized by resonance. Therefore it is reasonable to predict an S N2 mechanism.<br />

fig 1<br />

Note: in later chapters, especially in chapters 10 and 12, we will see many more reactions<br />

involving ATP. As we will see then, the SAM-forming reaction above is somewhat<br />

unusual – most reactions with ATP involve nucleophilic attack at one <strong>of</strong> the phosphate<br />

groups, rather than at the methylene (CH 2) carbon.<br />

E9.2:


<strong>Chapter</strong> 9<br />

Due to the relatively high nucleophilicity <strong>of</strong> the amine group, it does not need to be<br />

deprotonated prior to attack. The deprotonation step takes place after the nucleophilic<br />

attack, when an ammonium ion is the proton donor (an ammonium ion is a relative strong<br />

acid and does not require a strong base for deprotonation). The alcohol, in contrast, is less<br />

nucleophilic and needs to be deprotonated by a strong base before acting as a nucleophile<br />

in an S N2 reaction.<br />

fig 1<br />

E9.3:<br />

The nucleophile in this reaction is an amine, as opposed to a thiolate in the protein<br />

farnesyltransferase reaction. A thiolate is a very powerful nucleophile, an amine<br />

somewhat less so. Therefore the N-alkylating reaction, with the weaker nucleophile, is<br />

likely to be more SN1 in character, and thus would be expected to exhibit higher<br />

sensitivity to fluorine substitution on the electrophilic substrate (in other words, more<br />

positive charge develops on C1 <strong>of</strong> the electrophile in the AMP reaction)<br />

E9.4:<br />

The S N1 model is a solvolysis reaction (water is the nucleophile), which as a rule proceed<br />

through carbocation intermediates. In the S N2 model the nucleophile is a relatively<br />

powerful iodide ion, a factor which favors a concerted mechanism.<br />

E9.5:<br />

Notice that in the second step, when water enters the picture, stereochemistry is<br />

conserved. If the step were indeed a direct nucleophilic displacement by water, the<br />

observed stereochemistry would be different: either inversion as show below in the case<br />

<strong>of</strong> an SN2 mechanism, or racemization in the case <strong>of</strong> an SN1 mechanism.<br />

89


fig 1<br />

E9.6:<br />

90<br />

<strong>Chapter</strong> 9<br />

This is essentially a nucleophilic (or ‘base-catalyzed’) ring-opening, with the nucleophilic<br />

thiol attacking the less hindered carbon <strong>of</strong> the epoxide. The right-side epoxide carbon is<br />

‘protected’ from nucleophilic attack by the negatively charged oxygens <strong>of</strong> the<br />

phosphonate group.<br />

End-<strong>of</strong>-chapter problems<br />

P9.1:<br />

a)


fig 2<br />

P9.2:<br />

(See Biochem 42, 1564)<br />

fig 3<br />

P9.3:<br />

<strong>Chapter</strong> 9<br />

a) Because the reaaction involves the transfer <strong>of</strong> a methyl group to an amine, the most<br />

likely biomolecule would be S-adenosylmethionine (SAM – see section 9.1).<br />

b) A mechanism with an abbreviated version <strong>of</strong> the SAM structure is shown below. This<br />

is an SN2 mechanism, similar to other SAM-dependent methyltransferase reactions.<br />

91


fig 3<br />

P9.4:<br />

92<br />

<strong>Chapter</strong> 9<br />

a) Because the process involves two S N2 displacements, it is reasonable to predict that it<br />

is the thiol group – the most nucleophilic group in glutathione – that is playing the key<br />

role. For this reason, we will use the abbreviation GSH for glutathione in the mechanism<br />

in part b) below.<br />

b)<br />

fig 3<br />

P9.5:<br />

(See Acc Chem. Res. 2001, 34, 681.) A likely mechanism could involve two successive<br />

S N2 methyl transfer steps, the first from 5-methyltetrahydr<strong>of</strong>olate to cobalamin, and the<br />

second from cobalamin to homocysteine, the direct precursor to methionine. This would<br />

account for the observed retention <strong>of</strong> configuration. (McMurry p. 282)


fig 3<br />

P9.6:<br />

Challenge problems<br />

<strong>Chapter</strong> 9<br />

C9.1: See Biochem 2005, 44, 8989; Structure 2004, 12, 927.<br />

c) (solvent isotope effect - H 2O deprotonation should not be rate-limiting if dissociative).<br />

C9.2: See Biochem 1983, 22, 806; Chem Res Toxicol 1997, 10, 2 (scheme 1).<br />

C9.3: See Chem Res Toxicol 1997, 10, 2 (fig 13)<br />

C9.4: See Biochem 43, 7187 (look at compound #6 just above the Materials and Methods<br />

section):<br />

C9.5: Silverman p. 261<br />

a,b) See J. Am. Chem. Soc. 1994, 116, 11594.<br />

c) See J. Am. Chem. Soc. 1987, 109, 7530.<br />

93


In-chapter exercises<br />

E10.1:<br />

94<br />

<strong>Chapter</strong> 10<br />

<strong>Chapter</strong> 10 <strong>solutions</strong><br />

We’ll use methyl phosphate as a simple example <strong>of</strong> a phosphate monoester. The three<br />

major resonance forms show how the three non-bridging oxygens share a double bond.<br />

Another way <strong>of</strong> putting this is that each <strong>of</strong> these bonds is a single σ bond plus 1/3 <strong>of</strong> a π<br />

bond – a bonding order <strong>of</strong> 1.33. However, if we also consider the minor resonance form<br />

in which the bridging oxygen is double-bonded (it's a minor form because <strong>of</strong> the<br />

separation <strong>of</strong> charge), we can also think <strong>of</strong> the double bond being shared, in a small part,<br />

by the bridging oxygen. We can approximate these ideas by saying that the bridging bond<br />

order is slightly more than 1, and the non-bridging bond order is slightly less than 1.33.<br />

O<br />

O P<br />

fig 1<br />

O<br />

O<br />

O CH 3 O P<br />

O CH 3<br />

O<br />

major resonance contributors<br />

O<br />

O P<br />

E10.2: figs for problem in solnfig01a<br />

O<br />

O CH 3<br />

minor resonance contributor<br />

O<br />

O P<br />

O<br />

O CH 3<br />

A,E, and I are all representations <strong>of</strong> an alkyl- or acyl-adenosine phosphate ester (the<br />

aspartyl adenosine phosphate species in the first figure <strong>of</strong> section 10.2E is an actual<br />

example <strong>of</strong> an acyl-adenosine phosphate ester)


<strong>Chapter</strong> 10<br />

F and N are both representations <strong>of</strong> an organic monophosphate ester, such as glucose-6phosphate<br />

(section 10.2B).<br />

C and G are both representations <strong>of</strong> an organic diphosphate ester, such as mevalonate 5diphosphate<br />

(section 10.2C).<br />

B, H, K, R, and S are all representations <strong>of</strong> adenosine triphosphate (ATP – section<br />

10.2A).<br />

D, L and O are all representations <strong>of</strong> inorganic diphosphate (also known as inorganic<br />

pyrophosphate).<br />

J, M, P, and Q are all representations <strong>of</strong> adenosine diphosphate (ADP).<br />

E10.3:<br />

We can use a number <strong>of</strong> alternative abbreviations for ATP and ADP in showing this<br />

mechanism, but the ones used below are relatively compact and still show the chemistry<br />

taking place at the γ-phosphate <strong>of</strong> ATP:<br />

fig 1b<br />

E10.4:<br />

B:<br />

H<br />

protein<br />

O<br />

O<br />

O P<br />

O<br />

O<br />

O P<br />

O<br />

O<br />

O AMP +<br />

O<br />

protein<br />

O<br />

P<br />

O<br />

O<br />

O P<br />

O<br />

O AMP<br />

The phosphorylation reaction does not involve any bonds to the two carbon stereocenters,<br />

so we do not expect any changes to stereochemical configuration.<br />

fig 1<br />

E10.5:<br />

OH<br />

NH<br />

O<br />

a threonine residue<br />

O O<br />

P<br />

O O<br />

NH<br />

O<br />

a phosphorylated threonine residue<br />

95


fig 1b<br />

96<br />

O O<br />

H 3N CO 2<br />

O<br />

O P<br />

O<br />

End-<strong>of</strong>-chapter problems<br />

O<br />

O P<br />

P10.1 (McMurry p. 195, step 5)<br />

fig 1<br />

ribose-A<br />

ribose-A<br />

ribose-A<br />

O<br />

O<br />

P<br />

O<br />

O<br />

O P<br />

α<br />

O<br />

O<br />

O<br />

O P O O<br />

O<br />

O<br />

O AMP + O P<br />

O<br />

β<br />

O P<br />

ATP<br />

O<br />

P<br />

O<br />

O<br />

O<br />

δ −<br />

O O<br />

α β<br />

O P O P<br />

O<br />

ADP<br />

P10.2: (McMurry p. 287 step 6)<br />

O<br />

O<br />

O<br />

O<br />

γ<br />

P<br />

O<br />

O<br />

O<br />

O<br />

P<br />

O<br />

O<br />

O<br />

P<br />

O<br />

O<br />

O<br />

O<br />

H 3N CO 2<br />

O<br />

δ −<br />

O<br />

O<br />

OH<br />

O<br />

OH<br />

OH<br />

O<br />

O P<br />

O<br />

O<br />

O<br />

O P<br />

O<br />

O P<br />

Because no bonds to stereocenters are affected, the stereochemistry is unchanged.<br />

+<br />

O<br />

O<br />

O<br />

O<br />

<strong>Chapter</strong> 10<br />

O<br />

O AMP


fig 2<br />

ribose-A<br />

P10.3: McMurry p. 317 top<br />

fig 2<br />

P10.4:<br />

B:<br />

OPP<br />

HO<br />

OPP<br />

HO<br />

O<br />

O<br />

O<br />

OH<br />

O<br />

P<br />

O<br />

O<br />

H<br />

OH<br />

O<br />

ATP<br />

OPP<br />

+<br />

O<br />

P<br />

O<br />

O<br />

O<br />

O<br />

P<br />

O<br />

O P O<br />

O<br />

O<br />

O P<br />

O<br />

O<br />

O<br />

P<br />

O<br />

AMP<br />

H<br />

PO<br />

:B<br />

O P<br />

ATP<br />

O<br />

O<br />

O<br />

CO 2<br />

OH<br />

O ribose-A<br />

CO 2<br />

OH<br />

OH<br />

OH<br />

O ribose-A<br />

+ ADP<br />

<strong>Chapter</strong> 10<br />

97


98<br />

A H<br />

H 3N<br />

O<br />

O<br />

O<br />

P<br />

O<br />

O<br />

O<br />

fig 3<br />

P10.5: McMurry p. 318,<br />

a) (323 bottom)<br />

b)<br />

fig 3<br />

PPO<br />

B:<br />

ADP<br />

PO<br />

O<br />

OH<br />

O<br />

H<br />

O<br />

N<br />

O<br />

P<br />

O<br />

OH<br />

O<br />

OH<br />

O<br />

N<br />

O<br />

OH<br />

H<br />

O<br />

H<br />

N<br />

H<br />

O<br />

NH<br />

:B<br />

O P<br />

N<br />

O<br />

O<br />

H<br />

O<br />

:B<br />

ADP<br />

A H<br />

H 3N<br />

H 3N<br />

PO<br />

O P<br />

OH<br />

O<br />

O<br />

O<br />

O<br />

O<br />

O O<br />

+<br />

O<br />

O<br />

O<br />

P<br />

O<br />

H<br />

H<br />

OH<br />

<strong>Chapter</strong> 10<br />

:B<br />

O N<br />

PPO + ADP<br />

O<br />

O<br />

OH<br />

O<br />

OH<br />

OH<br />

O<br />

P<br />

O<br />

H<br />

N<br />

N<br />

OH<br />

O<br />

NH<br />

O<br />

N<br />

O<br />

P<br />

O<br />

O<br />

+ ADP


P10.6:<br />

<strong>Chapter</strong> 10<br />

This is a transfer <strong>of</strong> a phosphate group to a nitrogen rather than to an oxygen, but the<br />

mechanism is analogous.<br />

fig 4<br />

B:<br />

H<br />

N<br />

enz<br />

N<br />

HO<br />

HO<br />

P10.7: (McMurry p. 345)<br />

H 3N<br />

O<br />

O<br />

O<br />

O<br />

P<br />

O<br />

HO<br />

O<br />

O<br />

O P O<br />

O<br />

O P O<br />

O<br />

O P O<br />

O<br />

ribose-A<br />

O<br />

OH<br />

H A<br />

fig 4<br />

P10.8: (From J. Biol. Chem. 280, 10774).<br />

HO<br />

HO<br />

fig 4<br />

OH<br />

HO<br />

O<br />

O<br />

O<br />

P<br />

O<br />

O<br />

O<br />

O P O<br />

O<br />

O P O<br />

O<br />

O P O<br />

O<br />

ribose-C<br />

P10.9: (From Biochemistry 2000, 39, 8603.)<br />

H 3N<br />

HO<br />

HO<br />

O<br />

N<br />

enz<br />

O<br />

OH<br />

HO<br />

O<br />

O<br />

O<br />

P<br />

N<br />

O<br />

O<br />

P<br />

O<br />

O<br />

+<br />

AMP<br />

HO<br />

HO<br />

+<br />

+<br />

O<br />

O P<br />

O<br />

OH<br />

HO<br />

O<br />

O P O<br />

O<br />

O P O<br />

O<br />

O<br />

O<br />

O P O<br />

O<br />

O ribose-C<br />

OH<br />

O P O<br />

O (PPi)<br />

99


100<br />

<strong>Chapter</strong> 10<br />

a) The stereochemistry <strong>of</strong> the substitution and location <strong>of</strong> the 18 O label in the product<br />

strongly suggests that this is an SN (probably SN1-like) displacement at the anomeric<br />

carbon atom, rather than attack by the water molecule at a phosphorus.<br />

fig 5<br />

HO<br />

HO<br />

OH<br />

HO<br />

O<br />

O<br />

O<br />

P<br />

O<br />

O<br />

O P<br />

O<br />

O ribose-G<br />

HO<br />

HO<br />

HO<br />

HO<br />

OH<br />

HO<br />

OH<br />

HO<br />

O<br />

O<br />

+<br />

18<br />

OH<br />

O<br />

O<br />

P<br />

O<br />

18<br />

O<br />

H<br />

H<br />

O<br />

O P<br />

O<br />

:B<br />

O<br />

ribose-G<br />

b) If water were to attack at the β-phosphorus <strong>of</strong> the GDP group, the expected product<br />

would be:<br />

HO<br />

HO<br />

OH<br />

HO<br />

O<br />

OH<br />

+<br />

O<br />

18 O<br />

P<br />

O<br />

O<br />

O P<br />

O<br />

O<br />

ribose-G<br />

fig 5<br />

Notice the different stereochemistry at the anomeric carbon, and the different location <strong>of</strong><br />

the 18 O label.<br />

P10.10: (From Biochemistry 44, 11476.)


Tyr<br />

fig 5<br />

DNA<br />

5'<br />

O<br />

O H<br />

B:<br />

O<br />

O<br />

P<br />

O<br />

3'<br />

O<br />

O<br />

5'<br />

O<br />

DNA<br />

Base n<br />

H A<br />

O<br />

3'<br />

Base n+1<br />

knicking<br />

re-ligating<br />

DNA<br />

O<br />

5'<br />

O<br />

O<br />

O<br />

P<br />

O<br />

3'<br />

O<br />

O<br />

5'<br />

<strong>Chapter</strong> 10<br />

O<br />

DNA<br />

Base n<br />

P10.11: (From Biochem 41, 9279). Using radiolabelled water, we could determine the<br />

course <strong>of</strong> the reaction. In the first case, the AMP would be radioactive, in the second<br />

case the sugar would be radioactive.<br />

H<br />

ribose-A<br />

O<br />

*<br />

ribose-A<br />

fig 5<br />

H<br />

O<br />

O<br />

P10.12:<br />

O<br />

P<br />

O<br />

O<br />

P<br />

O<br />

:B<br />

O<br />

O<br />

O<br />

P<br />

O<br />

H<br />

O<br />

P<br />

O<br />

O<br />

O<br />

*<br />

O<br />

HO<br />

H<br />

HO<br />

O OH<br />

:B<br />

OH<br />

O OH<br />

OH<br />

ribose-A<br />

ribose-A<br />

O<br />

O<br />

O<br />

P<br />

O<br />

O<br />

P<br />

O<br />

O*<br />

O<br />

A H<br />

+<br />

Tyr<br />

+<br />

O<br />

*O<br />

O<br />

P<br />

O<br />

O<br />

P<br />

O<br />

O<br />

H<br />

O<br />

HO<br />

HO<br />

:B<br />

O<br />

3'<br />

O OH<br />

OH<br />

Base n+1<br />

O OH<br />

OH<br />

101


attack at γ-phosphate:<br />

102<br />

Nu:<br />

O<br />

O P O<br />

O<br />

attack at β-phosphate:<br />

Nu:<br />

O<br />

O P O<br />

O<br />

O<br />

O P O<br />

O<br />

attack at α-phosphate:<br />

O P O<br />

O<br />

O<br />

O<br />

O P O<br />

O<br />

Nu:<br />

O<br />

O<br />

P<br />

O<br />

P<br />

O<br />

O P<br />

O<br />

O P<br />

O<br />

O<br />

O<br />

O<br />

P<br />

O<br />

ATP<br />

O<br />

P<br />

O<br />

O<br />

P<br />

O<br />

O<br />

O P<br />

O<br />

O<br />

O P<br />

O<br />

O<br />

O P<br />

O<br />

O ribose-A<br />

O ribose-A<br />

eg. phosphoribosyl diphosphate (PRPP), section 10.2C<br />

O ribose-A<br />

:Nu<br />

O ribose-A<br />

:Nu<br />

O ribose-A<br />

Nu<br />

O<br />

P<br />

O<br />

O<br />

+ ADP<br />

eg. glucose-6-kinase, section 10.2B<br />

O<br />

O P O<br />

Nu<br />

O<br />

(no examples so far - but look at UTP<br />

reaction in section 11.4A)<br />

(no examples in text)<br />

O<br />

O P O<br />

O<br />

O<br />

P<br />

O<br />

+<br />

+ AMP<br />

O<br />

P<br />

O<br />

O<br />

O P<br />

HO ribose-A<br />

O<br />

O<br />

O<br />

P<br />

O<br />

O<br />

O P<br />

+ Pi<br />

O<br />

Nu<br />

Nu<br />

Nu P O ribose-A + PPi<br />

O<br />

eg. generation <strong>of</strong> acyl adenoside phosphate, section 10.2E<br />

<strong>Chapter</strong> 10<br />

O ribose-A


attack at 5'-carbon:<br />

O<br />

O P O<br />

O<br />

O<br />

P<br />

O<br />

O<br />

O P<br />

O<br />

:Nu<br />

O O<br />

N<br />

N<br />

HO OH<br />

NH 2<br />

N<br />

N<br />

Nu ribose-A + PPPi<br />

eg. excercise 9.1<br />

fig 6<br />

Here’s an example <strong>of</strong> this last possibility (the solution to exercise E9.1):<br />

Challenge Problems<br />

C10.1: See Biochem J. 1982, 201, 665.<br />

C10.2: See J. Biol. Chem. 257, 14795.<br />

C10.3: See J. Mol. Biol. 1999, 286, 1507.McMurry p. 176<br />

<strong>Chapter</strong> 10<br />

103


In-chapter exercises<br />

104<br />

<strong>Chapter</strong> 11<br />

<strong>Chapter</strong> 11 <strong>solutions</strong><br />

E11.1: The β-phosphate <strong>of</strong> UTP is attacked by the nucleophilic hydroxyl group, and<br />

inorganic phosphate is expelled. This is the first such phosphoryl transfer reaction that we<br />

have seen (we have seen the β-phosphate <strong>of</strong> ATP attacked by an alcohol in section 10.2C,<br />

but that attack was from the 'other side' - it resulted in expulsion <strong>of</strong> AMP and formation <strong>of</strong><br />

an organic diphosphate).<br />

E11.2:<br />

a) cycic ketal formation:<br />

fig 1<br />

I<br />

I<br />

I<br />

H A<br />

O<br />

O O<br />

O O<br />

H<br />

:B<br />

I<br />

I<br />

O<br />

H<br />

HO OH<br />

O<br />

OH<br />

B:<br />

I<br />

I<br />

H<br />

O<br />

O<br />

OH<br />

OH<br />

OH<br />

OH 2<br />

H A


) cyclic ketal hydrolysis:<br />

I<br />

I<br />

I<br />

fig 1<br />

E11.3:<br />

O O<br />

O<br />

HO OH<br />

O<br />

H<br />

H A<br />

:B<br />

I<br />

I<br />

HO<br />

O O H<br />

OH<br />

O H<br />

A<br />

H<br />

I<br />

I<br />

O<br />

O<br />

OH<br />

H 2O<br />

OH<br />

O<br />

H<br />

H<br />

:B<br />

<strong>Chapter</strong> 11<br />

105


106<br />

B:<br />

fig 1a<br />

R<br />

H<br />

PO<br />

N<br />

H<br />

CO 2<br />

H<br />

PO<br />

R<br />

N<br />

N<br />

N<br />

H A<br />

OH<br />

CO 2<br />

N<br />

CH 3<br />

End-<strong>of</strong>-chapter problems<br />

OH<br />

CH 3<br />

P11.1: (McMurry p. 250 bottom)<br />

fig 2<br />

enz<br />

O 2C O<br />

A H<br />

H 2N<br />

enamine form<br />

O<br />

H<br />

O<br />

O 2C O<br />

O<br />

:B<br />

B:<br />

PO<br />

B:<br />

R<br />

H<br />

H<br />

PO<br />

N<br />

R<br />

H<br />

CO 2<br />

N<br />

N<br />

H<br />

N<br />

CO 2<br />

N<br />

A H<br />

OH<br />

CH 3<br />

H A<br />

H 2N<br />

OH<br />

CH 3<br />

NH 2<br />

O 2C O<br />

imine form<br />

H<br />

O H<br />

O<br />

:B<br />

O 2C O<br />

B:<br />

NH 3<br />

O<br />

H<br />

O<br />

enz<br />

<strong>Chapter</strong> 11<br />

enz


P11.2: (McMurry p. 255 bottom, 2nd step)<br />

fig 2<br />

B:<br />

H<br />

O<br />

H<br />

N<br />

H<br />

H<br />

A<br />

CO 2<br />

P11.3: (McMurry p. 320, step 10)<br />

PO N<br />

O<br />

HO<br />

fig 2<br />

Mechanism:<br />

N<br />

OH<br />

O<br />

N<br />

H<br />

NH 2<br />

O<br />

B:<br />

H<br />

O<br />

H 2O<br />

H<br />

N<br />

H<br />

CO 2<br />

PO N<br />

O<br />

HO<br />

O<br />

O<br />

N<br />

H<br />

N<br />

H<br />

:B<br />

N<br />

N N<br />

H<br />

O<br />

N N<br />

ribose-5-P<br />

H A<br />

ribose-5-P<br />

B:<br />

H<br />

fig 3<br />

P11.4: (McMurry p. 307)<br />

IMP<br />

N<br />

N<br />

OH<br />

H<br />

OH<br />

H<br />

A<br />

O<br />

O<br />

N<br />

N<br />

H<br />

NH 3<br />

IMP<br />

<strong>Chapter</strong> 11<br />

CO 2<br />

107


fig 3<br />

108<br />

A H<br />

O<br />

N<br />

NH 2<br />

N<br />

R<br />

Cytidine<br />

P11.5: (McMurry p. 311 middle)<br />

H<br />

O H<br />

R<br />

N<br />

N<br />

N<br />

:B<br />

NH 2<br />

H<br />

O<br />

fig 3<br />

P11.6: (McMurry p. 264 step 1<br />

Lys<br />

O<br />

H<br />

N H<br />

O<br />

:B<br />

O<br />

N<br />

O<br />

H<br />

O<br />

A<br />

H<br />

H<br />

A<br />

:B<br />

R<br />

H<br />

N<br />

N<br />

B:<br />

O<br />

O<br />

H<br />

O<br />

N<br />

Lys<br />

O<br />

H<br />

H 2N<br />

N<br />

NH 2<br />

N<br />

R<br />

NH<br />

N<br />

OH<br />

O<br />

H<br />

HN<br />

H<br />

OH<br />

O<br />

A<br />

O<br />

N<br />

R<br />

Uridine<br />

A<br />

O<br />

H<br />

A<br />

R<br />

R<br />

N<br />

N<br />

H<br />

H<br />

N<br />

N<br />

H<br />

O<br />

A<br />

H3N O<br />

N<br />

:B<br />

N<br />

O<br />

N<br />

N<br />

R<br />

O<br />

O H<br />

NH 3<br />

NH<br />

NH<br />

O<br />

H<br />

Lys<br />

<strong>Chapter</strong> 11<br />

:B<br />

A<br />

N<br />

O<br />

O


fig 3<br />

P11.7: (McMurry p. 265 2nd step)<br />

O<br />

fig 4<br />

O<br />

O<br />

O<br />

N<br />

H<br />

H<br />

O H<br />

H A<br />

:B<br />

P11.8: (McMurry p. 277 step 4)<br />

O 2C<br />

B:<br />

fig 4<br />

O<br />

H<br />

OH<br />

N<br />

H<br />

P11.9: (McMurry p. 277 step 7)<br />

fig 5<br />

O<br />

O<br />

NH 3<br />

O<br />

O2C N CO2 P11.10 (McMurry p. 287 step 2)<br />

O<br />

O<br />

O<br />

O<br />

O<br />

O2C HO<br />

N<br />

H<br />

O<br />

O<br />

A H<br />

H 2O<br />

OH<br />

O<br />

O 2C<br />

O<br />

NH 3<br />

O<br />

O<br />

+<br />

O<br />

H 2N<br />

O<br />

N<br />

H H<br />

A<br />

O H<br />

H<br />

O<br />

CO 2<br />

O<br />

:B<br />

NH 3<br />

<strong>Chapter</strong> 11<br />

O<br />

OH<br />

N<br />

NH 3<br />

O<br />

O<br />

O<br />

O<br />

O<br />

109


fig 5<br />

110<br />

H<br />

HO<br />

CO 2<br />

O<br />

H<br />

OH<br />

OH<br />

OP<br />

P11.11 (McMurry p. 288 step iv)<br />

fig 5<br />

H 2C<br />

HO<br />

O<br />

OH<br />

CO 2<br />

OH<br />

reverse <strong>of</strong> hemiketal<br />

formation<br />

H 2C<br />

OH<br />

P11.12: (McMurry p. 291 bottom first step)<br />

fig 5<br />

OP<br />

A H<br />

O<br />

HO OH<br />

H<br />

N<br />

P11.13: (J. Biol. Chem. 280, 13712)<br />

CO 2<br />

OH<br />

OP<br />

O<br />

HO<br />

O<br />

OH<br />

OH<br />

CO 2<br />

OP<br />

CO 2<br />

OH<br />

anomeric carbon<br />

enol-keto<br />

tautomerization<br />

HO HO<br />

OP<br />

OH N<br />

H<br />

H<br />

OH N<br />

HO OH<br />

H<br />

H 3C<br />

:B<br />

O<br />

CO 2<br />

CO 2<br />

OH<br />

<strong>Chapter</strong> 11<br />

O<br />

OH<br />

CO 2


fig 6<br />

A H<br />

O<br />

C<br />

H H<br />

H<br />

N<br />

N<br />

H<br />

H<br />

:B<br />

N R<br />

B:<br />

A H<br />

P11.14: (J. Biol. Chem. 280, 12858, scheme 2 part 2)<br />

B:<br />

fig 6<br />

A H<br />

P11.15:<br />

HO<br />

O<br />

H<br />

H<br />

H<br />

O<br />

N<br />

O<br />

H<br />

O<br />

N<br />

O O<br />

R<br />

O<br />

O<br />

hemiketal<br />

formation<br />

HO<br />

O<br />

R<br />

H<br />

N<br />

N<br />

H<br />

OH<br />

N R<br />

O<br />

HO<br />

HN<br />

H<br />

N<br />

H 2C<br />

R<br />

N<br />

N<br />

H<br />

HN<br />

HO<br />

HN<br />

A H N<br />

N<br />

H<br />

O<br />

H<br />

O<br />

O O<br />

OH O<br />

O<br />

H<br />

H<br />

O<br />

hydrate<br />

formation<br />

HO<br />

R<br />

O<br />

O<br />

H<br />

<strong>Chapter</strong> 11<br />

N<br />

H<br />

R<br />

O<br />

O O<br />

:B<br />

OH<br />

OH OH<br />

O<br />

111


Challenge problems<br />

C11.1: (McMurry p. 296 step 5)<br />

1)<br />

2)<br />

112<br />

H 2N<br />

N<br />

H<br />

O<br />

N<br />

O<br />

N<br />

R 1<br />

N NH<br />

O<br />

R 2<br />

R 2<br />

NH<br />

A<br />

H<br />

H<br />

N<br />

H<br />

H<br />

:B<br />

H 2O<br />

imine formation<br />

H 2N<br />

N<br />

O<br />

N<br />

R 1<br />

H<br />

N NH<br />

O<br />

H<br />

N H<br />

C11.2: See Biochemistry 1994, 33, 13792, mechanism II.<br />

C11.3: See Biochemistry 2000, 39, 8603.<br />

C11.4: See J. Am. Chem. Soc. 2005, 127, 16412.<br />

H<br />

A<br />

N<br />

R 2<br />

N<br />

R 2<br />

H<br />

:B<br />

:B<br />

H 2N<br />

N<br />

tautomerization<br />

O<br />

N<br />

R 1<br />

HN<br />

NH 2<br />

R 2<br />

N<br />

<strong>Chapter</strong> 11<br />

+<br />

O<br />

NH<br />

R 2<br />

NH


In-chapter exercises<br />

E12.1:<br />

a) carboxylic acid<br />

b) thiol<br />

c) water<br />

<strong>Chapter</strong> 12<br />

<strong>Chapter</strong> 12 <strong>solutions</strong><br />

E12.2: Amines are bases, and thus the amine would simply deprotonate the carboxylic<br />

acid rather than attacking the carbonyl carbon.<br />

E12.3:<br />

O<br />

OH<br />

SOCl 2<br />

O<br />

R NH2 O C H<br />

R<br />

E12.4: Excess acetic acid is used in order to drive the equilibrium towards product<br />

formation (by Le Chatelier’s principle).<br />

O<br />

E12.5: Base-catalyzed esterification (starting with a carboxylic and an alcohol) will not<br />

work. The base will simply deprotonate the carboxylic acid, and carboxylate groups are<br />

very unreactive towards acyl substitution reactions.<br />

O<br />

R O<br />

Cl<br />

H<br />

+ R OH X no reaction<br />

E12.6: This reaction would be described as the acid-catalyzed hydrolysis <strong>of</strong> an ester.<br />

N<br />

O<br />

N<br />

113


H 3C C O<br />

114<br />

O<br />

E12.7:<br />

H 3C<br />

H 3C<br />

H<br />

O<br />

O<br />

O<br />

OH<br />

H<br />

O<br />

OH +<br />

OH<br />

O<br />

+<br />

+<br />

OH<br />

O<br />

S<br />

O<br />

H 3C<br />

HO<br />

+<br />

HO<br />

HO<br />

OH<br />

O<br />

H<br />

O<br />

+<br />

CH 3CH 2OH<br />

O<br />

H<br />

H 3C C O<br />

H 2O<br />

HO<br />

H 2SO 4<br />

H 2SO 4<br />

H 2SO 4<br />

H 2SO 4<br />

O<br />

H3C O<br />

propyl acetate<br />

(pear)<br />

O<br />

H 3C O<br />

benzyl acetate<br />

(peach)<br />

O<br />

H O<br />

isobutyl formate<br />

(raspberry)<br />

O<br />

O<br />

ethyl butanoate<br />

(pineapple)<br />

H<br />

O<br />

H3C C O<br />

O<br />

H H<br />

H<br />

O<br />

H3C C O<br />

O<br />

H H<br />

O<br />

O<br />

S<br />

O<br />

<strong>Chapter</strong> 12<br />

OH


OH<br />

O<br />

OH<br />

+<br />

CH 3OH<br />

H 2SO 4<br />

OH<br />

O<br />

O<br />

methyl salicylate<br />

(wintergreen)<br />

CH 3<br />

<strong>Chapter</strong> 12<br />

E12.8: The leaving group in this acyl transfer reaction is a resonance-stabilized phenolate<br />

ion - the negative charge is delocalized to the carboxylate group.<br />

O<br />

E12.9:<br />

O<br />

E12.10:<br />

a)<br />

H 3C<br />

O<br />

O CH 3<br />

O<br />

O<br />

O<br />

C SCoA<br />

enzyme<br />

+<br />

HS<br />

O<br />

Ph O<br />

enz<br />

HN<br />

O<br />

enz<br />

O<br />

HN<br />

O<br />

H<br />

O<br />

O<br />

S<br />

O<br />

O<br />

CO 2<br />

+<br />

H 3C<br />

O<br />

O<br />

enzyme<br />

115


)<br />

O<br />

116<br />

O HO<br />

CH 3<br />

O<br />

H<br />

End-<strong>of</strong>-chapter problems<br />

P12.1: A thioester (acetyl-CoA) is converted to an amide in this reaction:<br />

(Biochem 44, 16275)<br />

H3C O<br />

C<br />

SCoA<br />

H<br />

:B<br />

H3C O<br />

C<br />

H<br />

SCoA<br />

A<br />

H3C R HN R HN<br />

R HN<br />

O<br />

HO<br />

HO<br />

NH2 O<br />

HO<br />

HO<br />

NH2 O<br />

HO<br />

HO<br />

P12.2: (McMurry p. 191)<br />

O<br />

O 2C SCoA<br />

O<br />

P12.3: (McMurry p. 203?? step 3)<br />

HO<br />

HO<br />

OP<br />

HO<br />

O<br />

O<br />

P<br />

O<br />

O<br />

O<br />

:B<br />

H H<br />

O<br />

HO<br />

HO<br />

O OP<br />

O 2C SCoA<br />

OP<br />

HO<br />

O<br />

H A<br />

O<br />

H<br />

O<br />

<strong>Chapter</strong> 12<br />

O<br />

O<br />

NH 2<br />

O 2C OP<br />

HO<br />

HO<br />

OP<br />

HO<br />

OH<br />

O<br />

O


P12.4: (McMurry p. 229 top)<br />

O<br />

HO OP<br />

O<br />

HO NH 2<br />

O<br />

PO NH 2<br />

A B C<br />

<strong>Chapter</strong> 12<br />

P12.5: This reaction can be described as the hydrolysis <strong>of</strong> an amide. (McMurry p. 244<br />

step 2)<br />

O 2C<br />

NH 3<br />

H<br />

O<br />

O<br />

N<br />

H<br />

O<br />

H 2O O<br />

H O<br />

P12.6: The second step <strong>of</strong> this epoxide-opening reaction is hydrolysis <strong>of</strong> the ester formed<br />

in the first step. The bond to the chiral carbon is not broken, thus stereochemistry is not<br />

inverted. Notice also that the oxygen from the active site aspartate residue is incorporated<br />

into the product.<br />

R<br />

6<br />

CO 2<br />

O 2C<br />

NH 3<br />

H 2N<br />

O O OH<br />

HO<br />

O O OH<br />

O<br />

:B<br />

H A<br />

H<br />

H<br />

Asp Asp Asp<br />

H O<br />

B:<br />

P12.7: (Silverman p. 263, J. Am. Chem. Soc. 115, 10466)<br />

a) Yes, the fact that the 18 O label from the active site aspartate is incorporated into the<br />

product indicates that this residue is the ring-opening nucleophile as in the mechanism<br />

above.<br />

R<br />

b) In a hypothetical alternate mechanism, water could open the ring directly. This would<br />

result in an unlabeled product. If the water attacked at the more substituted side <strong>of</strong> the<br />

epoxide, as shown below, the result would be the enantiomer <strong>of</strong> the product that was<br />

actually observed by the investigators.<br />

6<br />

CO 2<br />

R<br />

OH<br />

O<br />

6<br />

O<br />

CO 2<br />

117


B:<br />

118<br />

H<br />

*<br />

O<br />

O<br />

*<br />

O<br />

H H<br />

O<br />

Asp<br />

P12.8: (McMurry p. 277 step 9)<br />

O 2C<br />

H<br />

O<br />

H<br />

O<br />

N<br />

H<br />

:B<br />

CO 2<br />

P12.9: (Stryer 6 p. 623)<br />

H 3C<br />

CH 3<br />

N<br />

CH 3<br />

O<br />

R C SCoA<br />

H<br />

O<br />

:B<br />

CO 2<br />

H 3N CO 2<br />

H 3C<br />

P12.10: (McMurry p. 323 bottom)<br />

O<br />

N<br />

O<br />

R<br />

CH 3<br />

CH 3<br />

O<br />

B:<br />

O 2C<br />

O 2C<br />

C SCoA<br />

O<br />

H<br />

H A<br />

CO 2<br />

HO<br />

O<br />

O<br />

O<br />

N<br />

H<br />

O<br />

OH<br />

H A<br />

CO 2<br />

( 18 O label stays on Asp)<br />

+<br />

H 3C<br />

H 3N CO 2<br />

H 2N<br />

CH 3<br />

N<br />

CH 3<br />

CO 2<br />

O<br />

<strong>Chapter</strong> 12<br />

O<br />

O<br />

H 3N CO 2<br />

O<br />

C R<br />

CO 2<br />

fatty acyl-carnetine


H N<br />

R<br />

NH<br />

H<br />

N<br />

R<br />

O<br />

N<br />

N<br />

:B<br />

H<br />

O<br />

O<br />

P<br />

O<br />

NH 2<br />

O<br />

ADP<br />

R<br />

H N<br />

N<br />

N<br />

R<br />

NH<br />

H<br />

NH<br />

N<br />

H A<br />

:B<br />

OP<br />

H<br />

=<br />

B:<br />

PO<br />

PO<br />

R<br />

N<br />

<strong>Chapter</strong> 12<br />

H A<br />

H N NH<br />

P12.11: (McMurry p. 308 step 3; crystal structure/mech in J. Biol. Chem. 281, 13762,<br />

scheme 2)<br />

a)<br />

H<br />

b)<br />

H<br />

H<br />

O<br />

H<br />

O<br />

:B<br />

:B<br />

O<br />

N<br />

H<br />

O<br />

N<br />

H<br />

N<br />

N<br />

H<br />

O<br />

H<br />

O<br />

B:<br />

O<br />

N<br />

H<br />

H<br />

N<br />

O<br />

O<br />

b<br />

H A<br />

H<br />

O<br />

N<br />

H<br />

O<br />

H<br />

N<br />

H<br />

O<br />

H A<br />

a a<br />

:B<br />

b<br />

H A<br />

O<br />

CO2 NH2 N<br />

H<br />

N CO 2<br />

H<br />

O<br />

NH 2<br />

NH 2<br />

N<br />

H<br />

CO 2<br />

O<br />

N<br />

R<br />

H<br />

N<br />

:B<br />

NH<br />

119


120<br />

<strong>Chapter</strong> 12<br />

c) Very <strong>of</strong>ten, enzymes activate a carbonyl for nucleophilic attack by coordinating a<br />

metal cation (usually Zn 2+ or Mg 2+ ) to the carbonyl oxygen, thus withdrawing electron<br />

density and increasing the electrophilicity <strong>of</strong> the carbon.<br />

O<br />

N<br />

H<br />

In this particular case, x-ray crystal structures show that two bound Zn 2+ ions participate<br />

in catalysis and help to direct nucleophilic attack at the proper carbonyl (see J. Biol.<br />

Chem. 2006, 281, 13762, scheme 2 for details).<br />

P12.12: (McMurry p. 296, step 3)<br />

A<br />

H<br />

H<br />

H<br />

O<br />

HN<br />

:B<br />

R<br />

N<br />

N<br />

N<br />

N<br />

R<br />

P12.13: (McMurry p. 318)<br />

B:<br />

H<br />

O<br />

H 2N<br />

R<br />

N<br />

H A<br />

N<br />

N<br />

N<br />

N<br />

H<br />

O<br />

R<br />

O<br />

N<br />

NH 2<br />

N<br />

N<br />

HN<br />

R<br />

R


B:<br />

H<br />

O<br />

N<br />

O<br />

O<br />

O<br />

N<br />

R<br />

O<br />

P<br />

UTP<br />

O ADP<br />

P12.14: (McMurry p. 322)<br />

OP<br />

OP<br />

O<br />

HO OH<br />

O<br />

H<br />

N<br />

HO OH<br />

H<br />

N<br />

O<br />

O<br />

glycinamide ribonucleotide<br />

O<br />

N<br />

H<br />

N<br />

H<br />

H A<br />

N<br />

H<br />

formylglycinamide ribonucleotide<br />

O<br />

OP<br />

N<br />

R<br />

H<br />

+<br />

H<br />

N<br />

H<br />

H<br />

H 2N<br />

H<br />

:B<br />

N<br />

H<br />

H 2N<br />

O<br />

B:<br />

H<br />

N<br />

N N<br />

H<br />

R<br />

PO<br />

H<br />

N<br />

O<br />

O<br />

10-formyl-THF<br />

N<br />

O<br />

N N<br />

THF<br />

P12.15: (McMurry p. 320 step 9; J. Biol. Chem 280, 10881 for archaea)<br />

H<br />

N<br />

H<br />

N<br />

R<br />

N<br />

N<br />

R<br />

NH 2<br />

N<br />

R<br />

CTP<br />

O<br />

NH 2<br />

H<br />

N H<br />

<strong>Chapter</strong> 12<br />

121


122<br />

<strong>Chapter</strong> 12<br />

The formate molecule is first converted to an acyl phosphate at the expense <strong>of</strong> one ATP.<br />

The formate is now activated for an acyl transfer reaction.<br />

N<br />

O<br />

H O<br />

R<br />

O<br />

N<br />

H<br />

O<br />

NH 2<br />

H<br />

:B<br />

O<br />

P<br />

O<br />

O ADP<br />

O<br />

H OP<br />

O<br />

H OP<br />

P12.16: (McMurry p. 323 top)<br />

H<br />

B:<br />

N<br />

H<br />

ribose-P<br />

N<br />

ribose-P<br />

NH<br />

O<br />

O<br />

P<br />

O<br />

O H<br />

H<br />

N H<br />

O<br />

O ADP<br />

N H<br />

O<br />

A H<br />

P12.17: (McMurry p. 320 step 7, p. 326)<br />

B:<br />

H<br />

N<br />

N<br />

R<br />

ribose-P<br />

N<br />

H<br />

O<br />

O<br />

N<br />

H<br />

OP H<br />

NH 2<br />

H<br />

OP<br />

H N H<br />

H H<br />

PO N H<br />

ribose-P<br />

N H<br />

:B<br />

O<br />

N H<br />

O<br />

N<br />

O<br />

R O<br />

NH<br />

NH 2<br />

H


R<br />

N<br />

N<br />

A<br />

P12.18:<br />

O<br />

NH 2<br />

O<br />

H<br />

C C N<br />

H<br />

H O H<br />

C C NH 3<br />

H O<br />

H<br />

H 2O<br />

H<br />

C<br />

H<br />

H<br />

H O<br />

H O<br />

OH<br />

C<br />

R<br />

O2C HN<br />

P12.19: (McMurry BA p. 665)<br />

P12.20:<br />

O<br />

H<br />

H<br />

N<br />

N<br />

H<br />

B<br />

N<br />

N<br />

CO 2<br />

H<br />

H<br />

C C NH<br />

H<br />

H<br />

O H<br />

H O H<br />

C C NH 2<br />

H O<br />

H H<br />

H 2O<br />

H O H H<br />

C C NH<br />

H<br />

H O H<br />

C C NH 2<br />

H<br />

H 2O<br />

<strong>Chapter</strong> 12<br />

H<br />

H O<br />

H<br />

123


H 3C C O<br />

O<br />

S<br />

O<br />

124<br />

O<br />

Cl<br />

+<br />

H<br />

Cl<br />

H<br />

O<br />

H 3C C<br />

O<br />

O<br />

H 3C C<br />

S<br />

Cl<br />

Cl<br />

Cl<br />

The PCl 3 mechanism:<br />

H 3C C OH<br />

fig 12<br />

O<br />

Cl<br />

P<br />

Cl<br />

Cl<br />

Challenge problems<br />

+ HCl<br />

C12.1: (McMurry p. 191)<br />

O<br />

H<br />

H 3C C O<br />

HO<br />

Cl<br />

H 3C C O<br />

H 3C C O<br />

Cl<br />

O<br />

H<br />

2 more times . . .<br />

O<br />

S<br />

P<br />

Cl<br />

Cl<br />

O<br />

S<br />

Cl<br />

Cl<br />

Cl<br />

OH<br />

P<br />

Cl<br />

H 3C C O<br />

O<br />

H 3C C O<br />

Cl<br />

O<br />

H 3C C O<br />

Cl<br />

Cl<br />

O<br />

+<br />

H<br />

P<br />

O<br />

S<br />

H<br />

Cl<br />

Cl<br />

H<br />

Cl<br />

Cl<br />

O<br />

O<br />

S<br />

H 3C C Cl<br />

<strong>Chapter</strong> 12<br />

Cl


CoAS<br />

O<br />

O<br />

O<br />

P<br />

O<br />

enz<br />

H<br />

O<br />

P<br />

O<br />

O<br />

O<br />

N<br />

O P<br />

GTP<br />

+<br />

O<br />

O<br />

N<br />

O<br />

O<br />

O<br />

GMP<br />

H A<br />

CoAS<br />

O<br />

O<br />

O P O<br />

A<br />

O<br />

H<br />

enz<br />

N<br />

O<br />

B:<br />

O<br />

O<br />

N P<br />

enz<br />

O<br />

H<br />

O<br />

O<br />

N<br />

O<br />

P<br />

O<br />

O<br />

N<br />

O<br />

O<br />

P<br />

O<br />

O<br />

O<br />

O<br />

O<br />

succinate<br />

GMP<br />

<strong>Chapter</strong> 12<br />

C12.2: (See J. Biol. Chem 1968, 243, 853 for experimental details). (Silverman p. 69)<br />

a) The cysteine could attack the 14 C atom, with subsequent loss <strong>of</strong> N 2 gas (this would be a<br />

very entropically favored step - we will see similar reaction types when we study<br />

decarboxylation mechanisms). This process would result in the 14 C label becoming<br />

attached to the active site cysteine.<br />

+<br />

O<br />

O<br />

O<br />

O<br />

125


enz<br />

126<br />

R<br />

S<br />

O<br />

H<br />

*<br />

C<br />

H<br />

N N<br />

:B<br />

enz<br />

R<br />

O<br />

N N<br />

*<br />

C<br />

S H<br />

enz<br />

R<br />

active site cysteine is<br />

radioactively labeled<br />

enz<br />

O<br />

*<br />

C +<br />

S H<br />

R<br />

O<br />

H<br />

*<br />

C<br />

S H<br />

H A<br />

<strong>Chapter</strong> 12<br />

b) The cysteine could also attack the carbonyl carbon, in what is essentially an acyl transfer<br />

reaction. In this case, the cysteine would not receive the radioactive 14 C label.<br />

enz<br />

S<br />

R<br />

O<br />

H<br />

*<br />

C<br />

H<br />

:B<br />

N N<br />

R<br />

enz<br />

O<br />

S<br />

active site cysteine is not<br />

radioactively labeled<br />

R<br />

*<br />

C<br />

H<br />

O<br />

H A<br />

N N<br />

S<br />

enz<br />

+<br />

R<br />

enz<br />

O<br />

S<br />

N N H<br />

C*<br />

H<br />

N N<br />

H<br />

C*<br />

H<br />

C12.3: See J. Biol. Chem 2000, 275, 40804 (Scheme 1 for the mechanism in part a, Scheme<br />

2 for part b)<br />

N N<br />

C12.4: (See also Biochemistry 2001, 40, 6989, Scheme 2) (McMurry p. 313, 314)<br />

Condensation step:


B:<br />

H 2N O<br />

H<br />

H<br />

O<br />

O<br />

P<br />

O<br />

O<br />

CO 2<br />

N CO 2<br />

aspartate<br />

Cyclization step:<br />

Zn 2+<br />

O<br />

O 2C<br />

Zn 2+<br />

O<br />

N<br />

H<br />

O<br />

N<br />

H<br />

carbanoyl aspartate<br />

H<br />

:B<br />

O 2C<br />

O 2C<br />

O 2C<br />

H 2N<br />

N<br />

H<br />

O<br />

O<br />

Zn 2+ Zn 2+ H A<br />

O<br />

N<br />

H<br />

O<br />

N<br />

H<br />

O<br />

O<br />

P<br />

O<br />

O<br />

O 2C<br />

H<br />

O<br />

N<br />

O<br />

N<br />

H<br />

=<br />

O<br />

N<br />

H<br />

N<br />

O 2C<br />

O 2C<br />

dihydroorotate<br />

N<br />

H<br />

<strong>Chapter</strong> 12<br />

O<br />

NH 2<br />

carbanoyl aspartate<br />

H<br />

O<br />

(flipped horizontally)<br />

CO 2<br />

127


In-chapter exercises<br />

E13.1:<br />

fig 1<br />

E13.2:<br />

128<br />

a)<br />

OH<br />

d)<br />

OH<br />

<strong>Chapter</strong> 13<br />

<strong>Chapter</strong> 13 <strong>solutions</strong><br />

OH<br />

b) c)<br />

H 2C C H<br />

There are many possibilities: if there is no α-proton present, there is no enol form<br />

possible. Here are three such compounds:<br />

E13.3:<br />

fig 1<br />

fig 1<br />

O<br />

H H<br />

OP<br />

O<br />

OH<br />

and<br />

OH<br />

OH<br />

OH<br />

OH<br />

O<br />

F 3C CF 3<br />

OH<br />

OH


<strong>Chapter</strong> 13<br />

E13.4: Propose a likely mechanism for glutamate racemase, showing stereochemistry<br />

throughout.<br />

fig 1<br />

E13.5:<br />

a)<br />

fig 2<br />

b)<br />

fig 2<br />

B:<br />

OP<br />

H<br />

H 3N<br />

OH<br />

OP<br />

OH<br />

O<br />

A H<br />

OH<br />

OH<br />

O<br />

CO 2<br />

O<br />

OH<br />

O OH<br />

H<br />

OH<br />

:B<br />

H 2O<br />

OP<br />

H 3N<br />

OH<br />

OP<br />

OP<br />

H A<br />

OH<br />

OH<br />

A H<br />

OH<br />

O<br />

OH<br />

O<br />

OH<br />

OH<br />

N<br />

CO 2<br />

O<br />

enz<br />

H H<br />

:B<br />

OH<br />

OH OH<br />

O H<br />

H A<br />

OH<br />

:B<br />

H 2O<br />

H 3N<br />

OP<br />

OP<br />

OH<br />

H<br />

O<br />

OP<br />

CO 2<br />

O<br />

OH<br />

H<br />

OH<br />

OH<br />

OH<br />

enz<br />

OH<br />

N<br />

A H<br />

N<br />

OH<br />

H H H<br />

H<br />

enz<br />

OH OH<br />

O<br />

O<br />

OH<br />

OH<br />

129


E13.6:<br />

fig 2<br />

E13.7:<br />

fig 2<br />

E13.8:<br />

fig 3<br />

E13.9:<br />

fig 3<br />

E13.10:<br />

130<br />

PO<br />

N<br />

H<br />

O<br />

H H<br />

B:<br />

H<br />

OH<br />

2<br />

O<br />

A H<br />

O<br />

OP<br />

PO<br />

OH<br />

OH<br />

O<br />

OP<br />

O<br />

OH<br />

O<br />

O<br />

O<br />

CH 2<br />

B:<br />

N<br />

H<br />

CO 2<br />

OP<br />

O 2C CO 2<br />

O O<br />

N<br />

H<br />

OH<br />

O<br />

H<br />

Lys<br />

H<br />

O<br />

:B<br />

OH<br />

OP<br />

OH<br />

OP<br />

O<br />

OH<br />

OH<br />

O<br />

O<br />

O<br />

OP<br />

O<br />

O<br />

O<br />

OH<br />

OH<br />

OH<br />

OP<br />

+<br />

OP<br />

OH<br />

O<br />

O<br />

N<br />

H<br />

<strong>Chapter</strong> 13<br />

H A<br />

OH<br />

OH<br />

OP


fig 3<br />

E13.11:<br />

a)<br />

b)<br />

A<br />

A<br />

A<br />

H<br />

O<br />

H<br />

O<br />

H<br />

O<br />

A<br />

O<br />

O<br />

H<br />

O<br />

O<br />

O<br />

O<br />

O<br />

Lys<br />

N<br />

O<br />

O<br />

O<br />

CO 2<br />

CO 2<br />

O<br />

CO 2<br />

A<br />

O<br />

HO<br />

A<br />

H :B<br />

Lys<br />

NH<br />

H<br />

+<br />

O<br />

O<br />

C<br />

O<br />

O<br />

CO 2<br />

CO 2<br />

<strong>Chapter</strong> 13<br />

fig 3<br />

c) The relationship is tautomeric: A and B are keto forms <strong>of</strong> the actual product. Usually<br />

keto forms are more stable than enols, but in the case <strong>of</strong> phenols the enol form is aromatic<br />

and is thus much lower in energy.<br />

E13.12:<br />

H<br />

O<br />

H<br />

O<br />

CO 2<br />

A<br />

B<br />

131


O<br />

fig 8<br />

E13.13:<br />

132<br />

O O<br />

O<br />

H H<br />

H 3C C CH 3<br />

O N H<br />

H 2O H<br />

fig 8<br />

H 2O<br />

H 2O<br />

H A<br />

H3C C<br />

O<br />

H H<br />

H 3C C<br />

E 13.14:<br />

O<br />

N<br />

H<br />

O O<br />

O<br />

OCH 2CH 3<br />

O<br />

H<br />

HO N<br />

O O<br />

O H OH 2<br />

H 3C C CH 3<br />

O<br />

H 3C C<br />

H 2O<br />

O O<br />

O<br />

N<br />

O<br />

Br<br />

O O<br />

O<br />

N<br />

O<br />

H 3C C CH 2<br />

O<br />

H<br />

:B<br />

O O<br />

O<br />

N<br />

H 3C C<br />

O<br />

OH<br />

O O<br />

O<br />

N<br />

O<br />

O<br />

H<br />

H 3C C CH 2<br />

O<br />

CH 3<br />

Cl<br />

<strong>Chapter</strong> 13<br />

H 3C<br />

Cl<br />

OH<br />

C<br />

O


PPh 3<br />

A B<br />

End-<strong>of</strong>-chapter problems<br />

Ph<br />

Ph<br />

P<br />

Ph<br />

C<br />

O<br />

<strong>Chapter</strong> 13<br />

P13.1: Propose a mechanism for this early reaction in the biosynthesis <strong>of</strong> the isoprenoid<br />

building blocks: (McMurry p. 129)<br />

O<br />

H<br />

C SCoA<br />

H H<br />

:B<br />

A H<br />

enz<br />

O<br />

O<br />

S<br />

O<br />

O<br />

O<br />

HO CH 3<br />

H<br />

O<br />

HO CH 3<br />

:B<br />

SCoA<br />

P13.2: (McMurry p. 121)<br />

enz<br />

SCoA<br />

S<br />

O<br />

H A<br />

O<br />

enz<br />

B:<br />

O<br />

H 2C SCoA<br />

S<br />

O<br />

O<br />

HO CH 3<br />

H O H<br />

SCoA<br />

133


fig 4<br />

134<br />

O<br />

O<br />

C<br />

O<br />

N N<br />

S<br />

H<br />

R<br />

P13.3: (McMurry p. 238)<br />

O<br />

O S<br />

O<br />

O<br />

O<br />

fig 4<br />

P13.4: (McMurry p. 239)<br />

fig 4<br />

P13.5:<br />

O<br />

H 3N<br />

CH 3<br />

O<br />

a) McMurry p. 250<br />

O<br />

H SCoA<br />

:B<br />

A H<br />

O<br />

N N<br />

O<br />

S<br />

O<br />

O<br />

H 3N<br />

O<br />

O<br />

S<br />

+<br />

O<br />

H<br />

R<br />

H 2C<br />

A H<br />

CH 3<br />

SCoA<br />

O<br />

O<br />

SCoA<br />

O<br />

O<br />

O<br />

H H<br />

N N<br />

O<br />

S<br />

O<br />

C<br />

O<br />

R<br />

H 3C SCoA<br />

H 3N<br />

H 3N<br />

O<br />

O<br />

O<br />

H 2C SCoA<br />

O<br />

O<br />

H 3C<br />

O<br />

<strong>Chapter</strong> 13<br />

O<br />

O<br />

H A<br />

O


O<br />

A<br />

H<br />

O<br />

B:<br />

H 2N<br />

O<br />

H<br />

O<br />

O<br />

fig 5<br />

b) J. Bact. 192, 1249 (c also)<br />

B:<br />

H<br />

solnfig 11<br />

c)<br />

CoAS<br />

O<br />

solnfig 11<br />

H 3C<br />

O<br />

H<br />

O<br />

SCoA<br />

CO 2<br />

A H<br />

O 2C<br />

:B<br />

O 2C<br />

O<br />

OH O<br />

O<br />

O<br />

A<br />

O<br />

O<br />

O<br />

H<br />

CoAS CH 2<br />

O<br />

NH 2<br />

O<br />

SCoA<br />

H<br />

CoAS CH 3<br />

acetyl-CoA<br />

O<br />

O<br />

O<br />

O 2C<br />

H3C CO2 pyruvate<br />

A<br />

O 2C<br />

O<br />

OH<br />

A H<br />

O<br />

OH HO<br />

O<br />

SCoA<br />

SCoA<br />

<strong>Chapter</strong> 13<br />

NH 3<br />

H<br />

O<br />

H<br />

O<br />

H<br />

H<br />

A<br />

O<br />

:B<br />

:B<br />

135


P13.6: McMurry p. 258, bottom)<br />

fig 5<br />

136<br />

H<br />

:B<br />

O<br />

SCoA<br />

O<br />

P13.7: (McMurry p. 277 step 3)<br />

fig 5<br />

O 2C<br />

O<br />

H<br />

pyruvate<br />

:B<br />

O 2C<br />

C O<br />

P13.8: (McMurry p. 286 first step)<br />

H 2C SCoA<br />

H<br />

O<br />

:B<br />

HO<br />

O<br />

CO 2<br />

O<br />

O<br />

H 3N<br />

fig 5<br />

P13.9: (McMurry p. 258 fig 5.20 step 3).<br />

O<br />

O<br />

CO 2<br />

SCoA<br />

SCoA<br />

CO 2<br />

H A<br />

O<br />

H A<br />

O<br />

O<br />

O<br />

O 2C<br />

B:<br />

HO<br />

HO<br />

O<br />

CO 2<br />

CO 2<br />

OH<br />

H 3N<br />

H O<br />

O<br />

O O<br />

H<br />

O<br />

SCoA<br />

SCoA<br />

H<br />

<strong>Chapter</strong> 13<br />

H A<br />

SCoA<br />

O<br />

:B<br />

O


fig 5<br />

O<br />

O<br />

P13.10: (McMurry p. 198)<br />

fig 6<br />

O 2C<br />

O<br />

O P<br />

O<br />

O<br />

SCoA<br />

O GDP<br />

P13.11: (McMurry p. 308 step 3-4)<br />

fig 6<br />

O<br />

N<br />

H<br />

H<br />

NH<br />

H<br />

O<br />

O<br />

H 3N<br />

:B<br />

O<br />

O<br />

O<br />

N<br />

H<br />

O<br />

O<br />

NH<br />

O<br />

O<br />

P13.12: (McMurry p. 291 bottom)<br />

A<br />

H<br />

HSCoA<br />

H :B<br />

B:<br />

H 3N<br />

H<br />

H 2N<br />

O<br />

O<br />

SCoA<br />

OP<br />

+<br />

O 2C CH 2<br />

O<br />

O<br />

O<br />

NH 2<br />

NH<br />

O<br />

H A<br />

O<br />

SCoA<br />

+ GDP<br />

O<br />

<strong>Chapter</strong> 13<br />

NH<br />

H 2N H A<br />

H 2N<br />

O<br />

H<br />

NH 2<br />

O<br />

H<br />

137


fig 6<br />

138<br />

CO 2<br />

N<br />

H<br />

HO<br />

H<br />

:B<br />

OH<br />

OH<br />

OP<br />

P13.13: (McMurry p. 362 last step)<br />

fig 7<br />

P13.14:<br />

H 3C<br />

O<br />

O<br />

O<br />

O<br />

H :B<br />

N<br />

H A<br />

CH 3<br />

A<br />

H H<br />

H<br />

CO 2<br />

N<br />

HO<br />

H O<br />

H 3C<br />

O<br />

O<br />

O<br />

H<br />

OH<br />

CO 2<br />

N<br />

OP<br />

:B<br />

HO<br />

H O<br />

N<br />

<strong>Chapter</strong> 13<br />

a) This is a Wittig reaction, followed by a transesterification (J. Org Chem 1985, 50,<br />

3420)<br />

H 3CO<br />

O<br />

O<br />

O<br />

O<br />

CH 3<br />

Ph 3P CH 2<br />

THF, -100 o C<br />

O<br />

K 2CO 3<br />

methanol<br />

H 3CO<br />

CH 2<br />

O<br />

OH<br />

CH 3<br />

OH<br />

OP


<strong>Chapter</strong> 13<br />

P13:15: Reactions a and b are acetoacetic ester syntheses; c is a malonic ester synthesis<br />

(section 13.6A).<br />

O<br />

O<br />

HO<br />

a b c<br />

P13.16: Reaction a is an acetoacetic ester synthesis, b is a malonic ester synthesis, and c<br />

is a Stork enamine alkylation (section 13.6A)<br />

fig 9<br />

P13.17:<br />

O<br />

O Br<br />

O<br />

A B C D<br />

OH<br />

O<br />

O<br />

N<br />

E F G<br />

a) b) c)<br />

OH<br />

d)<br />

f)<br />

F 3C C CH 3<br />

fig 9<br />

P13.18:<br />

O<br />

A<br />

O<br />

N<br />

H<br />

F 3C<br />

CH 3<br />

O<br />

g)<br />

CH 3<br />

O<br />

e)<br />

O<br />

O<br />

Br<br />

O<br />

O<br />

H 3C N 3<br />

O<br />

Br<br />

139


a)<br />

fig 10<br />

c)<br />

140<br />

O<br />

fig 10<br />

Br<br />

Mg<br />

O<br />

N<br />

H<br />

TsOH<br />

1)<br />

2) H 3O +<br />

MgBr<br />

Br<br />

CO 2<br />

O<br />

H 3O +<br />

HO CH3<br />

C CH3<br />

O<br />

C<br />

H 2C Ph 3P<br />

OH<br />

H 3O +<br />

SOCl 2<br />

CH 2<br />

<strong>Chapter</strong> 13<br />

O<br />

C Cl<br />

CH 3MgBr<br />

(2 equiv)<br />

O CH3<br />

C CH3


Challenge problems<br />

C13.1: (McMurry p. 288, step iv-v)<br />

fig 7<br />

B:<br />

H<br />

O<br />

O<br />

OH<br />

CO 2<br />

OH<br />

O<br />

O<br />

OH<br />

CO 2<br />

=<br />

OH O<br />

O<br />

O<br />

HO<br />

H A<br />

OH<br />

CO 2<br />

OH<br />

CO 2<br />

OH<br />

OH<br />

<strong>Chapter</strong> 13<br />

141


In-chapter exercises<br />

E14.1:<br />

E14.2:<br />

a)<br />

142<br />

O<br />

O<br />

N<br />

Nu:<br />

NO 2<br />

F<br />

X<br />

O<br />

H<br />

:B<br />

N<br />

H<br />

R<br />

???<br />

R<br />

O<br />

<strong>Chapter</strong> 14<br />

<strong>Chapter</strong> 14 <strong>solutions</strong><br />

X<br />

electrons cannot be<br />

stabilized by resonance<br />

with the carbonyl group<br />

N<br />

O<br />

fig 1<br />

b) The second nitro group further stabilizes the negatively-charged intermediate (the<br />

negative charged can be delocalized to either one <strong>of</strong> the nitro groups).<br />

O<br />

O 2N<br />

NO 2<br />

F<br />

N<br />

H<br />

NO 2<br />

N<br />

H<br />

R<br />

R<br />

O<br />

O


O<br />

N<br />

O<br />

O<br />

N<br />

F<br />

N<br />

H<br />

O<br />

R<br />

O<br />

O<br />

N<br />

O<br />

N<br />

F<br />

N<br />

H<br />

R<br />

<strong>Chapter</strong> 14<br />

c) The step in which the halogen leaves (the second step) is NOT the rate-determining<br />

step in this reaction – the rds is the initial attack by the nucleophile (which temporarily<br />

disrupts the aromaticity <strong>of</strong> the ring). Thus the fact that fluorine is a relatively poor<br />

leaving group is inconsequential. A fluorine substituent actually works better than other<br />

halogens because it is more electronegative, making the attached carbon more electronpoor<br />

and thus a better nucleophile, increasing the rate <strong>of</strong> the rds.<br />

fig 1<br />

E14.3:<br />

B:<br />

Cys<br />

fig 1<br />

H<br />

E14.4:<br />

fig 2<br />

H<br />

S<br />

O<br />

HN<br />

O<br />

N<br />

OH<br />

H<br />

O<br />

N<br />

R<br />

O<br />

P<br />

OH<br />

OH<br />

A H<br />

Cys<br />

S<br />

HN<br />

O<br />

N<br />

OH<br />

OH 2<br />

N<br />

R<br />

O<br />

O<br />

HN<br />

O<br />

HN<br />

O<br />

N<br />

H<br />

O<br />

O<br />

N<br />

N<br />

R<br />

O<br />

B: H H A<br />

H<br />

H<br />

O<br />

O<br />

P<br />

OH<br />

N<br />

R<br />

OH<br />

143


The cyclohexyl phosphate could form if the phosphate attacked the carbocation<br />

intermediate as a nucleophile rather than as a base:<br />

E14.5:<br />

144<br />

H<br />

N<br />

H<br />

O<br />

O<br />

P<br />

OH<br />

OH<br />

H A<br />

O2C CO2 B:<br />

B:<br />

H<br />

H OH B:<br />

H NH2 H<br />

H CO2<br />

N<br />

PO H<br />

PO PO<br />

H<br />

R<br />

OH<br />

OH<br />

H<br />

H A<br />

O<br />

CH 3<br />

B: H A<br />

Lys<br />

H<br />

N<br />

H<br />

N<br />

H<br />

PO<br />

H H<br />

R<br />

OH<br />

H<br />

OP<br />

N<br />

H<br />

N<br />

H<br />

H<br />

N<br />

Enz-PLP<br />

OH<br />

Lys<br />

CH 3<br />

CH 3<br />

CO 2<br />

H<br />

B:<br />

Lys<br />

CO 2<br />

H 2N CO 2<br />

glutamate<br />

H<br />

NH 2<br />

O<br />

R<br />

O<br />

P<br />

OH<br />

H<br />

OP<br />

R<br />

OH<br />

N<br />

H<br />

N<br />

H<br />

H H<br />

N<br />

CH 3<br />

R<br />

OH<br />

CH 3<br />

CO 2<br />

H<br />

H<br />

H<br />

OP<br />

N<br />

H<br />

N<br />

H<br />

<strong>Chapter</strong> 14<br />

H<br />

H<br />

N<br />

N<br />

R<br />

OH<br />

CH 3<br />

R<br />

OH<br />

CH 3<br />

CO 2<br />

H A<br />

CO 2


fig 2<br />

E14.6: McMurry p. 280 step 3<br />

The elimination phase <strong>of</strong> the γ-substitution is:<br />

H<br />

O<br />

H 3N CO 2<br />

O<br />

OP<br />

CO 2<br />

Schiff base<br />

formation<br />

N<br />

H<br />

N<br />

OH<br />

R 1<br />

CH 3<br />

CO 2<br />

OP<br />

N<br />

H<br />

O R 1<br />

The addition phase completes the substitution:<br />

OP<br />

N<br />

H<br />

N<br />

O<br />

H<br />

B:<br />

CH 3<br />

CO 2<br />

H<br />

S<br />

CO 2<br />

NH 3<br />

O<br />

R 2<br />

OP<br />

B:<br />

N<br />

H<br />

N<br />

H<br />

N<br />

OH<br />

CH 3<br />

O<br />

CH 3<br />

O R 1<br />

CO 2<br />

CO 2<br />

O<br />

OP<br />

N<br />

H<br />

N<br />

OP<br />

H<br />

O<br />

CH 3<br />

N<br />

H<br />

CO 2<br />

B:<br />

N<br />

H<br />

H<br />

O<br />

O R 1<br />

S R2 A H<br />

A H<br />

H<br />

OP<br />

O<br />

N<br />

H<br />

H<br />

<strong>Chapter</strong> 14<br />

CH 3<br />

N<br />

O R 1<br />

CO 2<br />

OH<br />

CH 3<br />

O<br />

S R 2<br />

CO 2<br />

145


fig 3<br />

fig 3<br />

E14.7:<br />

H 3C<br />

146<br />

R<br />

H 3C<br />

R<br />

H 3C<br />

R<br />

fig 4<br />

E14.8:<br />

OP<br />

R<br />

N<br />

S<br />

R<br />

N<br />

S<br />

R<br />

N<br />

S<br />

H<br />

N<br />

H<br />

enz<br />

N<br />

A H<br />

H 3C<br />

O<br />

CH 3<br />

OH<br />

CH 3<br />

H<br />

O<br />

O<br />

+<br />

O<br />

:B<br />

H CH 3<br />

H<br />

S<br />

H 3N CO 2<br />

O<br />

cystathionine<br />

H 3C<br />

R<br />

R<br />

N<br />

S<br />

CO 2<br />

H 3C<br />

R<br />

NH 3<br />

R<br />

N<br />

S<br />

H A<br />

OH<br />

CH 3<br />

C O<br />

The mechanism for the hydrolysis reaction can be depicted:<br />

O<br />

OH<br />

CH3 O<br />

OP<br />

O<br />

H 3C<br />

R<br />

N<br />

H<br />

N<br />

R<br />

N<br />

S<br />

H<br />

<strong>Chapter</strong> 14<br />

OH<br />

CH 3<br />

S R 2<br />

CO 2<br />

OH<br />

CH3 O<br />

O


S<br />

R<br />

C<br />

S<br />

R<br />

H H<br />

O<br />

H<br />

HS SH +<br />

S<br />

R<br />

C<br />

S<br />

R<br />

O<br />

H H<br />

O<br />

H<br />

R R<br />

H<br />

H<br />

O<br />

H<br />

HS<br />

HS<br />

H<br />

H<br />

S O<br />

H<br />

C<br />

R R<br />

S<br />

R<br />

C<br />

H<br />

O<br />

R<br />

<strong>Chapter</strong> 14<br />

O H<br />

H<br />

O H<br />

The methyl iodide helps to prevent the reverse reaction (thioacetal formation) by serving<br />

as an electrophilic 'trap' for the thiol groups:<br />

I<br />

H<br />

C<br />

H<br />

H<br />

End-<strong>of</strong>-chapter problems<br />

HS SH<br />

H<br />

H<br />

P14.1: (See Biochemistry 1997, 36, 12526) (McMurry p. 177)<br />

Lys<br />

O<br />

NH 2<br />

O<br />

H<br />

OP<br />

OH<br />

=<br />

Mg 2+<br />

Lys<br />

O<br />

NH 3<br />

O<br />

H<br />

C<br />

OP<br />

I<br />

OH<br />

H<br />

O<br />

Glu<br />

O<br />

S S<br />

CH 3<br />

CH 3<br />

H<br />

147


148<br />

Lys<br />

NH 3<br />

O<br />

O<br />

O<br />

H H<br />

OP<br />

O<br />

Glu<br />

O<br />

<strong>Chapter</strong> 14<br />

P14.2: This reaction is an elimination followed by enamine to imine tautomerization<br />

followed by hydrolysis <strong>of</strong> the imine. (McMurry p. 239, fig 5.8 bottom path)<br />

H 3N H<br />

OH<br />

O<br />

:B<br />

O OH<br />

A H<br />

O<br />

O<br />

O<br />

fig 5<br />

P14.3: (McMurry p. 252 top)<br />

fig 5<br />

O 2C<br />

H 3N CO 2<br />

NH 3<br />

O<br />

H 3N<br />

H<br />

O<br />

A H<br />

O<br />

:B<br />

O<br />

NH 4<br />

O<br />

O 2C<br />

H<br />

A H<br />

A H<br />

H<br />

O<br />

H H<br />

CO 2<br />

aspartate fumarate<br />

P14.4: A likely mechanism for cysteine 'tagging' is by a Michael addition.Michael addition)<br />

N<br />

H<br />

O<br />

NH 2<br />

:B<br />

O<br />

O<br />

:B<br />

O


fig 6<br />

Cys<br />

S<br />

H<br />

:B<br />

O<br />

N<br />

O<br />

Cys<br />

A H<br />

P14.5: This is a dehydration followed by tautomerization. McMurry p. 285 bottom<br />

H 3C OH<br />

H 3C O<br />

HO<br />

H<br />

O<br />

:B<br />

fig 6<br />

P14.6: (McMurry p. 260 3rd step)<br />

O<br />

H H<br />

H 2C<br />

:B<br />

O<br />

O OH<br />

A<br />

P14.7: (McMurry p. 286)<br />

fig 6<br />

CO 2<br />

H 3C OH<br />

S<br />

H 3C O<br />

HO<br />

A<br />

CO 2<br />

H A<br />

OH<br />

H<br />

O<br />

O<br />

O<br />

O OH<br />

A<br />

N<br />

O<br />

CO 2<br />

intermediate product<br />

OH<br />

A<br />

CO 2<br />

Cys<br />

CH 3<br />

CH 3<br />

S<br />

O<br />

H 3C O<br />

O<br />

O<br />

<strong>Chapter</strong> 14<br />

O<br />

H 3C O<br />

H<br />

HO<br />

O H<br />

O<br />

N<br />

O<br />

O OH<br />

:B<br />

A<br />

149


150<br />

<strong>Chapter</strong> 14<br />

P14.8: This is likely an E1 mechanism, with a resonance-stabilized carbocation<br />

intermediate. Notice that the more common E1cb mechanism is not possible, as there is<br />

no adjacent carbonyl group. (McMurry p. 291)<br />

O<br />

CO 2<br />

CO 2 CO2<br />

NH 3<br />

H<br />

CH 2<br />

H A<br />

O<br />

CO 2<br />

NH 3<br />

H<br />

fig 6<br />

P14.9: This is an E1cb elimination - AMP is the leaving group. McMurry p. 326)<br />

R<br />

N<br />

N<br />

O 2C<br />

HN CO 2<br />

N<br />

N<br />

R<br />

N<br />

N<br />

CH 3<br />

N<br />

AMP<br />

:B<br />

NH 2<br />

N<br />

+<br />

O 2C<br />

CO 2<br />

NH 3<br />

CO 2<br />

fumarate<br />

fig 7<br />

P14.10: All <strong>of</strong> the following reactions are PLP-dependent. Draw mechanisms, showing<br />

the ‘electron sink’ action <strong>of</strong> PLP. In each case, begin the substrate-PLP adduct, and end<br />

with the product-PLP adduct (in other words, you do not need to show the Schiff base<br />

being formed and later hydrolyzed)<br />

a) This is a retro-aldol reaction. (McMurry p. 239 fig 5.8 middle path)


N<br />

H<br />

enz<br />

N<br />

PLP-enzyme<br />

OH<br />

CH 3<br />

enz<br />

H3N CO2 N glycine<br />

OP OP<br />

N<br />

H<br />

OP<br />

PLP-enzyme<br />

OH<br />

CH 3<br />

+<br />

H 3C<br />

OH<br />

H 3N CO 2<br />

threonine<br />

N<br />

H<br />

N<br />

OH<br />

CH 3<br />

CO 2<br />

OP<br />

OP<br />

A H<br />

N<br />

H<br />

H 3C O<br />

N<br />

H<br />

N<br />

OH<br />

CH 3<br />

PLP-threonine<br />

N<br />

OH<br />

CH 3<br />

CO 2<br />

H<br />

CO 2<br />

<strong>Chapter</strong> 14<br />

:B<br />

H 3C O<br />

H<br />

acetaldehyde<br />

fig 8<br />

b) This is something we haven't seen before - a PLP-dependent retro-Claisen. Notice that<br />

a carbon-carbon bond is broken. (McMurry p. 248)<br />

HO<br />

NH 2<br />

O<br />

R<br />

NH 3<br />

O<br />

O<br />

OP<br />

:B<br />

O2C H<br />

R<br />

N<br />

H<br />

N<br />

OH<br />

PLP-substrate<br />

O<br />

CH 3<br />

OP<br />

B:<br />

O 2C R<br />

N<br />

H<br />

N<br />

H<br />

OH<br />

O<br />

O<br />

CH 3<br />

H<br />

151


152<br />

OP<br />

fig 8-9<br />

OP<br />

O 2C<br />

N<br />

H<br />

O 2C CH 3<br />

N<br />

N<br />

H<br />

PLP-alanine<br />

N<br />

CH 3<br />

OH<br />

CH 3<br />

c) (McMurry p. 283)<br />

PO<br />

N<br />

H A<br />

OH<br />

CH 3<br />

OP<br />

O 2C<br />

N<br />

H<br />

OP<br />

N<br />

CH 2<br />

H<br />

O<br />

H A<br />

CH 3<br />

N<br />

H<br />

enz<br />

N<br />

PLP-enzyme<br />

O<br />

O R<br />

OH<br />

CH 3<br />

+<br />

OP<br />

O 2C<br />

H<br />

O<br />

O2C R<br />

N<br />

H<br />

NH 3<br />

alanine<br />

OP<br />

N CO2 OP<br />

N CO2 H<br />

OP<br />

OH<br />

O<br />

H<br />

PLP-substrate<br />

H<br />

CH 3<br />

:B<br />

N<br />

H<br />

H<br />

PO H<br />

CH 3<br />

:B<br />

N<br />

CH 3<br />

PO<br />

N<br />

H<br />

O<br />

H<br />

CH 3<br />

N<br />

H<br />

O<br />

<strong>Chapter</strong> 14<br />

:B<br />

CH 3<br />

O<br />

CO 2


fig 9<br />

P14.11:<br />

OP<br />

OP<br />

O O<br />

H<br />

fig 10<br />

H<br />

A H<br />

O<br />

N<br />

H<br />

N<br />

H<br />

N<br />

N<br />

OCH 3<br />

OH<br />

CH 3<br />

OH<br />

CH 3<br />

OH<br />

CO 2<br />

OH<br />

CO 2<br />

OP<br />

O O<br />

B:<br />

N<br />

H<br />

O<br />

N<br />

H O<br />

H H<br />

O<br />

OH<br />

CH 3<br />

CO 2<br />

A H<br />

OP<br />

N<br />

H<br />

N<br />

OH<br />

CH 3<br />

<strong>Chapter</strong> 14<br />

CO 2<br />

O O<br />

O O<br />

O O<br />

O O<br />

H OH 2<br />

153


P14.12: McMurry p. 284)<br />

H 3C<br />

154<br />

H 3C<br />

R<br />

H 3C<br />

R<br />

R<br />

(<br />

fig 10<br />

P14.13:<br />

R<br />

N<br />

S<br />

R<br />

N<br />

S<br />

R<br />

N<br />

A H<br />

S H3C O<br />

+<br />

H 3C<br />

H<br />

H 3C<br />

O<br />

CO2 OH<br />

:B<br />

O<br />

O<br />

O<br />

HO CO 2<br />

H 3C<br />

R<br />

A<br />

H<br />

R<br />

N<br />

S<br />

H 3C<br />

R<br />

O<br />

O<br />

R<br />

N<br />

S<br />

OH<br />

CH 3<br />

O<br />

OH<br />

CH3 O<br />

CO 2<br />

Br Br OH<br />

O<br />

H 3C<br />

R<br />

R<br />

N<br />

S<br />

<strong>Chapter</strong> 14<br />

OH<br />

CH3 O<br />

O


fig 12<br />

<strong>Chapter</strong> 14<br />

P14.14: Notice that in order for the (E)-alkene to form, the C-H and C-Br bonds must be<br />

anti and co-planar.<br />

B:<br />

H<br />

H<br />

H3C CH 3<br />

H<br />

Br<br />

H<br />

H<br />

H3C :B δ-<br />

CH 3<br />

H<br />

δ<br />

Br<br />

-<br />

B H<br />

H 3C H<br />

fig 12<br />

P14.15: Recall that for an E2 reaction to occur on a cyclohexane ring, the leaving group<br />

and the abstracted hydrogen must both be in axial positions.<br />

Ph<br />

Ph<br />

OTs<br />

three large groups axial - very unfavorable!<br />

P14.16:<br />

a)<br />

b)<br />

I<br />

+<br />

O<br />

O<br />

O CF 3<br />

dimethyl sulfoxide<br />

H 3C<br />

CH 3<br />

C O<br />

CH 3<br />

methanol<br />

H<br />

Br<br />

If bromine leaving group is axial,<br />

there are no axial hydrogens on<br />

neighboring carbons<br />

E2<br />

O<br />

S N2<br />

CH 3<br />

Br<br />

155


c)<br />

d)<br />

e)<br />

P14.17:<br />

H<br />

156<br />

O<br />

OH<br />

fig 14<br />

P14.18:<br />

H 3C<br />

Br<br />

OH<br />

OTs<br />

I<br />

OH<br />

O<br />

SH<br />

acetone<br />

O<br />

H<br />

H<br />

O<br />

OH<br />

F<br />

OH<br />

methanol<br />

methanol<br />

65 o C<br />

S N2<br />

O<br />

O<br />

SH<br />

HO H<br />

O<br />

OCH 3<br />

S N1<br />

O<br />

CH 3<br />

E1<br />

HO H<br />

O<br />

F<br />

+ enantiomer<br />

O<br />

O<br />

H<br />

O<br />

OH<br />

<strong>Chapter</strong> 14


a) b)<br />

fig 14<br />

P14.19:<br />

a)<br />

b)<br />

O<br />

O<br />

fig 14<br />

P14.20:<br />

O<br />

+<br />

O O<br />

O<br />

+<br />

O<br />

O<br />

O<br />

CH 3<br />

a) b) c)<br />

Challenge problems<br />

C14.1: (McMurry p. 317)<br />

O<br />

O<br />

O<br />

O<br />

O<br />

O<br />

O<br />

N(CH 3) 2<br />

+ C 2H 4<br />

<strong>Chapter</strong> 14<br />

a) It is a decarboxylation reaction, but there is no obvious way for the electron pair to be<br />

stabilized.<br />

fig 11<br />

O<br />

HN<br />

O<br />

N<br />

R<br />

???<br />

O<br />

O<br />

157


) A carbene intermediate would imply a very hydrophobic active site pocket – see<br />

Chemical and Engineering News, May 12, 1997, p. 12; Science 1997, 276, 942.<br />

c) See Chemical and Engineering News, March 13, 2000, p. 42.<br />

C14.2: See Biochemistry 2003, 42, 248 (scheme 1) McMurry p. 330)<br />

158<br />

<strong>Chapter</strong> 14<br />

C14.3: The first part is cyclic imine formation, then we see an E1cb dehydration adjacent<br />

to the imine. McMurry p. 277, steps 4,5)<br />

O 2C<br />

B:<br />

(fig 11<br />

C14.4:<br />

H A<br />

O OH<br />

H<br />

N<br />

H<br />

CO 2<br />

A<br />

H<br />

HO<br />

O2C N CO2 OH<br />

O2C N CO2 H<br />

O 2C<br />

H<br />

H<br />

H<br />

:B<br />

OH<br />

N<br />

H A<br />

OH<br />

O<br />

O2C N CO2 B:<br />

H<br />

O<br />

H A<br />

a) Lacking any other information, it is reasonable to expect a mechanism that parallels<br />

that <strong>of</strong> asparagine synthetase (section12.2B): conversion to an acyl phosphate, then acyl<br />

transfer with ammonia nucleophile. (McMurry p. 320 step 7-8)<br />

b) The nitrogen comes from the amino group on aspartate, and is freed by the final<br />

elimination step.


ibose-P<br />

fig 11<br />

O<br />

O<br />

NH 2<br />

ATP ADP<br />

O<br />

ribose-P<br />

OP<br />

NH 2<br />

aspartate<br />

P i<br />

E1cb<br />

ribose-P<br />

O<br />

ribose-P<br />

O<br />

O 2C<br />

NH<br />

NH 2<br />

N<br />

H<br />

NH 2<br />

<strong>Chapter</strong> 14<br />

CO 2<br />

CO 2<br />

CO 2<br />

159


In-chapter exercises<br />

E15.1:<br />

fig 1<br />

E15.2:<br />

fig 1<br />

E15.3:<br />

a)<br />

b)<br />

fig 1<br />

160<br />

δ +<br />

δ<br />

E<br />

+<br />

H Br :Br<br />

<strong>Chapter</strong> 15<br />

<strong>Chapter</strong> 15 <strong>solutions</strong><br />

Br<br />

H 3C OH<br />

δ<br />

Nu<br />

-<br />

δ +<br />

E<br />

Br


E15.4:<br />

<strong>Chapter</strong> 15<br />

a) The regiochemistry <strong>of</strong> both electrophilic addition steps is governed by the relative<br />

stabilities <strong>of</strong> the carbocation intermediates. Intermediate 1 is stabilized by<br />

hyperconjugation with the neighboring methyl group, while intermediate 2 is stabilized<br />

by resonance with the bromine.<br />

b)<br />

H3C C C H<br />

H Br<br />

H3C C C H<br />

Br<br />

H OH 2<br />

H 3C<br />

Br<br />

H 2O<br />

:Br<br />

H<br />

H3C C C<br />

slowest<br />

H<br />

step intermediate 1<br />

C CH 3<br />

CH 3<br />

H3C C C H<br />

C CH 3<br />

O<br />

H<br />

H 2O<br />

:Br<br />

H Br<br />

Br H<br />

C C<br />

H3C H<br />

Br<br />

H<br />

Br<br />

H<br />

C C H C C<br />

H3C H<br />

H3C H<br />

intermediate 2<br />

H 2O<br />

H3C C C H<br />

O H<br />

H H<br />

H3C C C H<br />

H<br />

O<br />

H<br />

H<br />

H OH 2<br />

c) Alkynes are less reactive towards electrophilic addition relative to alkenes. This is<br />

because the vinylic carbocation intermediate involved is very high energy. In the case <strong>of</strong><br />

HBr addition, for example, once the monobrominated alkene forms, the second HBr will<br />

add very rapidly.<br />

E15.5:<br />

H<br />

161


H OH 2<br />

fig 1<br />

E15.6:<br />

Br<br />

fig 1<br />

E15.7:<br />

162<br />

O<br />

OH<br />

O H OH 2<br />

R R<br />

O<br />

O<br />

H 2SO 4<br />

Br<br />

O<br />

O<br />

HO<br />

O<br />

R R<br />

O<br />

OH<br />

Mg O<br />

MgBr<br />

O<br />

H 2O<br />

O<br />

H<br />

H 3O +<br />

OH<br />

O<br />

<strong>Chapter</strong> 15<br />

OH<br />

R R<br />

This is an intramolecular reaction – thus there is inherently a much lower entropy<br />

component to the energy barrier. In other words, the electrophilic atom is held close to<br />

the aromatic π bonds, which makes initiation <strong>of</strong> the electrophilic attack easier.<br />

End-<strong>of</strong>-chapter problems<br />

P15.1: (McMurry p. 388)<br />

OH<br />

OH


fig 2<br />

R 1<br />

S<br />

CH 3<br />

R 2<br />

R<br />

R<br />

H 3C<br />

N<br />

N<br />

H R<br />

H<br />

P15.2: (McMurry p. 383 step 4)<br />

fig 2<br />

P15.3:<br />

O 2C<br />

B:<br />

R1 N<br />

N R2 HO<br />

O 2C<br />

H<br />

CO 2<br />

N<br />

H<br />

H<br />

A<br />

CO 2<br />

H<br />

CO 2<br />

R<br />

H<br />

CO 2<br />

R<br />

O 2C<br />

R<br />

R<br />

R<br />

O 2C<br />

O 2C<br />

H 3C<br />

H<br />

N<br />

R<br />

R<br />

<strong>Chapter</strong> 15<br />

R<br />

R1 N<br />

N R2 O 2C<br />

N<br />

H<br />

B:<br />

H<br />

+<br />

CO 2<br />

O<br />

H H<br />

CO 2<br />

N<br />

H<br />

H<br />

CO 2<br />

R<br />

CO 2<br />

163


O<br />

164<br />

mechanism A:<br />

O<br />

P<br />

18 O<br />

O 2C<br />

O<br />

mechanism B:<br />

O<br />

O<br />

P<br />

18 O<br />

O 2C<br />

O<br />

O 2C<br />

O<br />

O<br />

A H<br />

O<br />

A H<br />

OH<br />

OH<br />

OH<br />

OH<br />

OH<br />

OH<br />

OH<br />

OH<br />

OH<br />

OH<br />

OP<br />

OP<br />

OP<br />

B:<br />

H O<br />

B:<br />

H<br />

O<br />

18 O<br />

O<br />

H O<br />

O<br />

18 O<br />

P O<br />

O<br />

18 O<br />

CO 2<br />

CO 2<br />

P O<br />

H<br />

CO 2<br />

18 O<br />

OH<br />

OH<br />

OH<br />

OH<br />

P O<br />

OH<br />

OH<br />

OH<br />

CO 2<br />

fig 3<br />

c) Mechanism B: See Biochem. Biophys. Res. Commun. 1988, 157, 816.<br />

P15.4: (JBC 264, 2075) this exp not described<br />

a)<br />

B:<br />

O<br />

H<br />

O<br />

O<br />

OH<br />

OH<br />

OH<br />

OH<br />

OH<br />

OH<br />

OH<br />

OP<br />

OH<br />

OP<br />

OP<br />

OH<br />

<strong>Chapter</strong> 15<br />

OP


PPO<br />

OPP<br />

OPP<br />

<strong>Chapter</strong> 15<br />

b) If the labeled substrate shown below were to undergo a concerted reaction, the label<br />

would necessarily be found on the β (outside) phosphate group <strong>of</strong> the product. If, on the<br />

other hand, the label were actually to be observed on the α (inside) phosphate <strong>of</strong> the<br />

product, this would rule out a concerted mechanism.<br />

fig 4<br />

O<br />

O<br />

O P O<br />

O<br />

P<br />

O<br />

18<br />

O R<br />

P15.5: (Arch Biochem Biophys 2003, 417, 203)<br />

OPP<br />

fig 4<br />

P15.6: Fig 1, Microbiol. 151, 2199<br />

a)<br />

O<br />

O<br />

O H<br />

H<br />

O<br />

P<br />

O<br />

O<br />

P<br />

18<br />

O O no label here<br />

:B<br />

R<br />

O<br />

H A<br />

OH<br />

O H<br />

:B<br />

165


)<br />

166<br />

PPO<br />

N<br />

H<br />

R<br />

fig 5<br />

P15.7: JBC 264, 2075<br />

R<br />

N<br />

H<br />

a) (+)-bornyl diphosphate<br />

HN<br />

O<br />

O<br />

N<br />

H<br />

N<br />

H<br />

R<br />

H<br />

:B<br />

N<br />

H<br />

N<br />

H<br />

HN<br />

R<br />

H<br />

O<br />

O<br />

N<br />

H<br />

<strong>Chapter</strong> 15<br />

N<br />

H<br />

:B<br />

R


) (+)-sabinene.<br />

OPP<br />

OPP<br />

H<br />

OPP<br />

(+)-bornyl diphosphate<br />

(+)-sabinene<br />

OPP<br />

<strong>Chapter</strong> 15<br />

fig 6<br />

c) This is an anti-Markovnikov addition, because the secondary carbocation, rather than<br />

the tertiary carbocation, forms during the addition.<br />

P15.8: Essentially, the cysteine is 'tricked' into acting as a nucleophile rather than as a<br />

base. (See J. Am. Chem. Soc. 2005, 127, 17433 for a complete description <strong>of</strong> the<br />

experiments referred to here).<br />

H<br />

:B<br />

167


168<br />

PPO<br />

P15.9:<br />

fig 7<br />

P15.10:<br />

A<br />

H 3C<br />

H<br />

C C H<br />

H OH 2<br />

PPO<br />

PPO<br />

H 2O<br />

H 3C<br />

C C H<br />

H 3C<br />

H<br />

O<br />

S<br />

H<br />

enz<br />

C CH 3<br />

:B<br />

B:<br />

H 3C<br />

H 3C<br />

PPO<br />

H H<br />

O<br />

C C H<br />

H<br />

O H<br />

H 2O<br />

C CH 3<br />

<strong>Chapter</strong> 15<br />

S<br />

H OH 2<br />

The NMR data shows that the main product is from anti-Markovnikov addition <strong>of</strong> HBr.<br />

This regiochemistry is due to the electron-withdrawing effect <strong>of</strong> the carbonyl group,<br />

which makes the primary carbocation intermediate more stable than the secondary<br />

carbocation.<br />

O O O H C C C<br />

3.5 ppm (t)<br />

Br<br />

H<br />

H<br />

H<br />

O<br />

CH 3<br />

3.0 ppm (t)<br />

2.2 ppm (s)<br />

If the reaction were to proceed with Markovnikov regiochemistry, the NMR spectrum <strong>of</strong><br />

the product would look very different:<br />

O O O<br />

Br<br />

d, 3H<br />

q, 1H<br />

H 3C<br />

H<br />

O<br />

C C<br />

Br<br />

CH 3<br />

s, 3H


(see J. Chem. Educ. 1990, 67, 518 for more details on this experiment).<br />

P15.11: (Silverman p. 277 bottom, similar to McMurry p. 289)<br />

fig 7<br />

O2C C<br />

PO<br />

H 2C<br />

HO<br />

C<br />

CH 2<br />

O<br />

CO 2<br />

O<br />

H A<br />

OH<br />

HN<br />

P15.12: (Silverman p. 282)<br />

O<br />

O<br />

CH 3<br />

O2C C<br />

PO<br />

ribose-U<br />

CH 3<br />

B:<br />

H<br />

H 2C<br />

PO<br />

HO<br />

C<br />

B:<br />

O<br />

CO 2<br />

HO<br />

H<br />

O<br />

O<br />

OH<br />

HN<br />

O<br />

OH<br />

HN<br />

O<br />

O<br />

CH 3<br />

O<br />

O<br />

CH 3<br />

ribose-U<br />

ribose-U<br />

<strong>Chapter</strong> 15<br />

We would expect (and in fact we observe) alkylation to occur ortho to the phenol group,<br />

because the associated carbocation intermediate can be stabilized by resonance with the<br />

phenol oxygen.<br />

fig 8<br />

O<br />

OH<br />

O<br />

O<br />

R<br />

OH<br />

intermediate<br />

O<br />

product<br />

O<br />

R<br />

OH<br />

O<br />

R<br />

169


170<br />

<strong>Chapter</strong> 15<br />

P15.13: This is a methylation reaction, so it is reasonable to expect (and in fact it is the<br />

case) that SAM is the required c<strong>of</strong>actor. (McMurry p. 388).<br />

R 1<br />

SAM<br />

O 2C<br />

S R 2<br />

CH 3<br />

R 2<br />

CO 2<br />

NH<br />

R 1<br />

B:<br />

fig 8<br />

P15.14: (J. Biol. Chem. 279, 39389)<br />

fig 9<br />

OP<br />

HO<br />

O<br />

OPP<br />

OH<br />

OP<br />

HO<br />

O 2C<br />

H 3C<br />

H<br />

R 2<br />

CO 2<br />

NH<br />

resonance-stabilized<br />

cationic intermediate<br />

O<br />

OH<br />

NH 2<br />

CO 2<br />

R 1<br />

O<br />

O 2C<br />

H 3C<br />

NH 2<br />

O<br />

OH<br />

P15.15: See Science 1997, 277, 1815 for a detailed discussion <strong>of</strong> these reactions.<br />

McMurry p. 144<br />

a)<br />

OP<br />

HO<br />

OP<br />

HO<br />

O<br />

OH<br />

R 2<br />

O<br />

NH 2<br />

CO 2<br />

NH<br />

R 1


OPP<br />

common intermediate<br />

methyl shift<br />

H<br />

hydride shift<br />

:B<br />

H<br />

epi-arisolochene<br />

A H<br />

B:<br />

H<br />

<strong>Chapter</strong> 15<br />

fig 9<br />

b) The mechanism is same as in part a) up to the point after the hydride shift and before<br />

the methyl shift.<br />

common intermediate<br />

H<br />

:B<br />

vetispiradiene<br />

fig 10<br />

P15.16: This is an acyloin rearrangement (section 15.7C). (McMurry p. 137)<br />

171


172<br />

A H<br />

H 3C<br />

B:<br />

O<br />

2<br />

H<br />

3 4<br />

O<br />

OH<br />

OP<br />

HO<br />

H 3C<br />

OH<br />

OP<br />

4 =<br />

2 3<br />

O<br />

H<br />

O<br />

OH<br />

2<br />

3 4<br />

H 3C OH<br />

<strong>Chapter</strong> 15<br />

fig 10<br />

P15.17: A tyrosine in that position is able to play an additional role in stabilizing the<br />

carbocation intermediate, through a cation-π interaction.<br />

P15.18:<br />

a)<br />

fig 11<br />

b)<br />

H Cl<br />

Br<br />

OTs<br />

OH<br />

c) H<br />

H<br />

H<br />

Cl<br />

H 2O<br />

H<br />

H<br />

O<br />

O H<br />

H<br />

TsO<br />

Cl<br />

H<br />

OP


d)<br />

fig 12<br />

15.19:<br />

a)<br />

H Cl<br />

O<br />

HO<br />

OH (+ enantiomer)<br />

(4 diastereomers)<br />

c) d)<br />

e)<br />

g)<br />

i)<br />

Cl<br />

j)<br />

O<br />

(major)<br />

O<br />

Br<br />

O<br />

NO 2<br />

CH 3<br />

Br<br />

+<br />

HNO 3<br />

H 2SO 4<br />

+ enantiomer<br />

O<br />

O<br />

NO 2<br />

O<br />

Cl<br />

O<br />

CH 3<br />

NO 2<br />

h)<br />

Cl<br />

b)<br />

NaOH(aq)<br />

f)<br />

NO 2<br />

OCH 3<br />

NO 2<br />

H 3O +<br />

Cl<br />

O<br />

Br Br<br />

OH<br />

Cl<br />

OtBDMS<br />

NO 2<br />

<strong>Chapter</strong> 15<br />

173


k)<br />

P15.20:<br />

a)<br />

b)<br />

c)<br />

P15.21:<br />

a)<br />

174<br />

O<br />

O CH 3<br />

O<br />

Br 2<br />

FeBr 3<br />

Br 2<br />

FeBr 3<br />

SO 3<br />

H 2SO 4<br />

B 2H 6<br />

O<br />

O<br />

O CH 3<br />

SO 3H<br />

Mg CO 2<br />

O<br />

H Cl<br />

AlCl 3<br />

(dry ice)<br />

H 2O 2, NaOH<br />

H 2O<br />

+ enantiomer<br />

c) d)<br />

COOH<br />

COOH<br />

NaOH(aq)<br />

NaH<br />

SOCl 2<br />

O<br />

HOCH 2CH 2OH, H + Mg<br />

b)<br />

CH 3<br />

Ph<br />

H 3O +<br />

O<br />

O<br />

N H<br />

OH<br />

SO 3H<br />

H 3O +<br />

O<br />

H 3O +<br />

O<br />

O<br />

N<br />

<strong>Chapter</strong> 15<br />

O<br />

OH<br />

OH


P15.22:<br />

a)<br />

b)<br />

c)<br />

d)<br />

P15.23:<br />

a)<br />

b)<br />

c)<br />

O<br />

O<br />

+<br />

O 2N<br />

+<br />

+<br />

+<br />

CH 3<br />

O<br />

O<br />

N<br />

CH 3<br />

OCH 3<br />

CH 3<br />

Br<br />

O<br />

O<br />

CH 3<br />

NO 2<br />

CH 3<br />

OCH 3<br />

O<br />

Br<br />

N<br />

O<br />

CH 3<br />

<strong>Chapter</strong> 15<br />

175


d) Note that the initial product <strong>of</strong> Claisen rearrangement has lost aromaticity - but<br />

aromaticity is restored by a subsequent keto-enol tautomerization step.<br />

176<br />

O O<br />

Challenge problems<br />

H<br />

C15.1: get NMR data from Ted, include J if available. provide ref<br />

OH<br />

<strong>Chapter</strong> 15<br />

Addition can take place with either Markovnikov or anti-Markovnikov regiochemistry:<br />

H 2C<br />

CH 3<br />

H<br />

O<br />

Br<br />

OCH 3<br />

H 3C<br />

H 2C<br />

O<br />

CH 3<br />

H 3C<br />

H<br />

OCH 3<br />

O<br />

OCH 3<br />

H 3C<br />

H 3C<br />

O<br />

Br<br />

H 3C<br />

OCH 3<br />

Markovnikov product<br />

Br<br />

H 2C<br />

H<br />

O<br />

OCH 3<br />

anti-Markovnikov product<br />

NMR data shows that it is the anti-Markovnikov product that forms, due to the electronwithdrawing<br />

effect <strong>of</strong> the ester carbonyl. (The 1 H spectrum <strong>of</strong> the Markovnikov product<br />

would be expected to contain just two singlet signals.)<br />

Peak assignments are given below. Notice that the the H R and H S protons are<br />

diastereotopic (there is a stereocenter in the molecule) and have different chemical shifts.<br />

These signals show dd splitting because H R and H S are coupled to each other, and also to<br />

the proton at 2.3 ppm.


fig 12<br />

3.5 ppm<br />

3.6 ppm<br />

H S<br />

H R<br />

1.3 ppm<br />

Br<br />

C<br />

H 3C<br />

H<br />

O<br />

OCH 3<br />

2.3 ppm<br />

C15.2: See Biochem. Biophys. Acta 1998, 1384, 387.<br />

C15.3:<br />

a) See Nature 2003, 422, 185.<br />

b) See J. Am. Chem. Soc., 127 3577.<br />

C15.4: McMurry p. 294<br />

HO<br />

O<br />

CO 2<br />

chorismate<br />

CO 2<br />

CH 2<br />

fig 10<br />

C15.5: (Silverman p. 285)<br />

A H<br />

O<br />

prephenate<br />

O<br />

HO CO 2<br />

O<br />

3.7 ppm<br />

<strong>Chapter</strong> 15<br />

O<br />

phenylpyruvate<br />

CO 2<br />

177


A H<br />

178<br />

NH 3<br />

R 1 R 2 R 1 R 2<br />

N<br />

H<br />

NH 3<br />

R 1<br />

N<br />

H<br />

R 2<br />

fig 10<br />

C15.6: (McMurry p. 285)<br />

N<br />

H<br />

NH 3<br />

NH 3<br />

R 1 R 2 R 1<br />

N<br />

H<br />

N<br />

H<br />

B:<br />

H<br />

N<br />

H<br />

N<br />

H<br />

R 2<br />

R 1 R 2 R 1 R 2<br />

The condensation <strong>of</strong> two pyruvate molecules requires the participation <strong>of</strong> thiamine<br />

diphosphate coenzyme:<br />

H 3C<br />

R 2<br />

H 3C<br />

R 2<br />

N<br />

S<br />

R 1<br />

N<br />

S<br />

R 1<br />

A H<br />

H 3C<br />

+<br />

O<br />

H 3C<br />

O<br />

O<br />

O<br />

CO 2<br />

H 3C OH<br />

H 3C<br />

R 2<br />

The second step is an acyloin rearangement:<br />

A H<br />

H 3C<br />

O<br />

CO 2<br />

H3C O H<br />

fig 11<br />

C15.7: See J. Org. Chem. 2003, 68, 5433.<br />

B:<br />

N<br />

S<br />

H 3C<br />

R 2<br />

HO<br />

H 3C<br />

R 1<br />

CH 3<br />

CH 3<br />

OH<br />

O<br />

R 1<br />

O<br />

H 3C<br />

R 2<br />

:B<br />

N CH3 O<br />

H<br />

S CO2 H3C OH<br />

O<br />

CO 2<br />

=<br />

OH<br />

O<br />

A H<br />

N<br />

S<br />

CO 2<br />

R 1<br />

<strong>Chapter</strong> 15<br />

CH 3<br />

OH<br />

H 3C CO 2<br />

O


In-chapter exercises<br />

<strong>Chapter</strong> 16<br />

<strong>Chapter</strong> 16 <strong>solutions</strong><br />

E16.1: An aldol condensation is not a redox reaction. There is no transfer <strong>of</strong> electrons<br />

from one species to another, either in the form <strong>of</strong> a hydrogenation/dehydrogenation step<br />

or in terms <strong>of</strong> an insertion <strong>of</strong> an oxygen atom (or other electronegative atom).<br />

E16.2: Epoxidation reactions are redox transformations: both carbon atoms lose a bond<br />

to carbon and gain a bond to oxygen. The cyclase reaction, however, is not redox.<br />

E16.3: This is a transformation that is thermodynamically favorable, but the kinetics are<br />

very slow. The activation energy, in other words, is too high for the transformation to<br />

occur uncatalyzed at room temperature at a rate that is noticeable.<br />

E16.4: The anomeric carbon (in other words, the carbon that is the center for the<br />

hemiacetal) has two bonds to oxygen – thus it is in the carbonyl oxidation state. This<br />

becomes clear when we look at the straight-chain form <strong>of</strong> glucose, in which this carbon is<br />

an aldehyde.<br />

E16.5: Saturated fat is in a more highly reduced state, and therefore contains more<br />

electrons to transfer to oxygen and more energy to release when oxidized to CO 2 during<br />

respiration.<br />

E16.6: In grape fermentation, glucose is converted to two equivalents <strong>of</strong> ethanol and one<br />

<strong>of</strong> carbon dioxide (this is the bubbling that is observed). When wine turns to vinegar, the<br />

ethanol is being oxidized to acetic acid. Grape juice (full <strong>of</strong> sugar) contains the most<br />

calories, and vinegar the least.<br />

E16.7:<br />

a) For every molecule <strong>of</strong> substrate that is oxidized, one molecule <strong>of</strong> NADP + is reduced to<br />

NADPH. We can determine the concentration <strong>of</strong> NADPH formed by using its extinction<br />

coefficient, ε = 6290 M -1 cm -1 .<br />

179


180<br />

6290 M -1 cm -1 =<br />

.096<br />

(1 cm)(c)<br />

c = 1.53 x 10 -5 M<br />

<strong>Chapter</strong> 16<br />

So the amount <strong>of</strong> substrate oxidized is (1.53 x 10 -5 M)(1 x 10 -3 L) = 1.53 x 10 -8 moles.<br />

b) (100 x 10 -6 M) – (1.53 x 10 -5 M) = 8.47 x 10-5 M = 84.7 µM.<br />

c)<br />

E16.8:<br />

fig 5<br />

E16.9:<br />

a)<br />

(4.0 x 10-13 1.53 x 10<br />

moles enz) (300 sec)<br />

-8 moles substrate reacted<br />

= 128 reactions per second<br />

per enzyme molecule<br />

Br Br<br />

Br Br<br />

S S<br />

Cl<br />

H 2O<br />

Br<br />

Br<br />

HO<br />

HO<br />

Br<br />

OH<br />

H 2O<br />

Br<br />

O<br />

H<br />

+ enantiomer<br />

S<br />

S<br />

Br<br />

Cl<br />

H<br />

+ enantiomer


)<br />

S<br />

S<br />

protein<br />

protein<br />

fig 6<br />

H<br />

S<br />

H A<br />

:B<br />

OH<br />

OH<br />

End-<strong>of</strong>-chapter problems<br />

P16.1: (McMurry p. 179)<br />

A H<br />

SH<br />

HS<br />

H 3C CO 2<br />

H<br />

N<br />

R<br />

H<br />

O<br />

O<br />

protein<br />

H A<br />

protein<br />

NH 2<br />

S<br />

S<br />

OH<br />

S<br />

H<br />

OH<br />

:B<br />

HO<br />

H 3C<br />

H<br />

N<br />

R<br />

O<br />

O<br />

O<br />

NH 2<br />

HS<br />

HS<br />

protein<br />

protein<br />

<strong>Chapter</strong> 16<br />

S S<br />

fig 1<br />

b) The hydride adds to the front <strong>of</strong> pyruvate as drawn in the figure. The is the re face.<br />

fig 1<br />

P16.2:<br />

a) (McMurry p. 250)<br />

O<br />

H 3C CO 2<br />

H<br />

OH<br />

OH<br />

181


B:<br />

fig 1<br />

b)<br />

182<br />

H<br />

H<br />

O NH 3<br />

O H<br />

B:<br />

H<br />

H<br />

O<br />

S CoA<br />

fig 1<br />

P16.3: (McMurry p. 277 steps 1-2)<br />

O<br />

O<br />

H 3N<br />

O<br />

O<br />

O<br />

O<br />

R<br />

O O H<br />

H R<br />

:B<br />

H<br />

N<br />

R<br />

O<br />

NH 2<br />

CO2 H<br />

S<br />

S<br />

CoA<br />

CoA<br />

ATP ADP<br />

fig 1<br />

P16.4: (McMurry p. 277 steps 5-6)<br />

PO<br />

O<br />

H 3N<br />

O<br />

A<br />

CO 2<br />

NAD +<br />

O<br />

O<br />

NADPH<br />

O<br />

Pi<br />

NADP +<br />

<strong>Chapter</strong> 16<br />

O NH 3<br />

H<br />

O<br />

H<br />

N<br />

R<br />

H<br />

+<br />

O<br />

CO 2<br />

H NAD<br />

O<br />

H 3N<br />

B<br />

O<br />

NH 2<br />

O<br />

O<br />

O


fig 2<br />

P16.5:<br />

O<br />

HN<br />

O<br />

N<br />

H<br />

B:<br />

H<br />

OH<br />

O2C N CO2 H<br />

H<br />

CO 2<br />

H A<br />

:B<br />

P16.6: (McMurry p. 327)<br />

A<br />

H<br />

O2C N CO2 OH<br />

O2C N CO2 H<br />

O<br />

HN<br />

O<br />

N<br />

H<br />

N<br />

N<br />

R<br />

H<br />

FMN<br />

CO 2<br />

O<br />

N<br />

H A<br />

N<br />

A<br />

H<br />

O<br />

H NADP<br />

O2C N CO2 H<br />

H<br />

O2C N CO2 H :B<br />

O<br />

HN<br />

H<br />

N<br />

N<br />

R<br />

O<br />

N<br />

H<br />

+<br />

FMNH 2<br />

<strong>Chapter</strong> 16<br />

O<br />

N<br />

H<br />

CO 2<br />

N<br />

H<br />

O<br />

183


R<br />

184<br />

N<br />

N<br />

B:<br />

fig 2<br />

P16.7:<br />

O<br />

N<br />

R<br />

N<br />

N<br />

NH<br />

H<br />

H<br />

S<br />

O<br />

N<br />

H<br />

NH<br />

O<br />

enz<br />

R<br />

N<br />

N<br />

R<br />

O<br />

N<br />

N<br />

N<br />

NH<br />

S<br />

A<br />

H<br />

O<br />

H<br />

N<br />

NAD +<br />

NH<br />

enz<br />

O<br />

H :B<br />

R<br />

R<br />

N<br />

N<br />

N<br />

N<br />

O<br />

N<br />

O<br />

N<br />

NH<br />

H<br />

S<br />

NH<br />

S<br />

<strong>Chapter</strong> 16<br />

O H<br />

a) First an imine (Schiff base) linkage forms, then the imine is reduced to an amine by<br />

NADPH.(McMurry p. 264 step 1)<br />

O 2C CO 2<br />

B:<br />

H A<br />

O<br />

H N CO2<br />

H<br />

lysine<br />

NH 3<br />

A H<br />

Lys<br />

HO N<br />

H<br />

O 2C CO 2<br />

:B<br />

CO 2<br />

O 2C N H<br />

saccharopine<br />

O<br />

OH<br />

NH 3<br />

O<br />

O 2C N<br />

CO 2<br />

enz<br />

H NADP<br />

fig 3<br />

b) The amine formed in the previous reaction (part a) is oxidized to an imine (not the<br />

reverse <strong>of</strong> the reduction step in part a – it occurs on the other side <strong>of</strong> the molecule!).<br />

Hydrolysis <strong>of</strong> the imine results in the products. (McMurry p. 265)<br />

H<br />

:B<br />

enz<br />

H A<br />

H A<br />

Lys


CO 2<br />

NH 3<br />

O 2C N CO 2<br />

B:<br />

CO 2<br />

H<br />

H<br />

NAD +<br />

+<br />

O<br />

NH3<br />

O2C NH2 CO2<br />

glutamate<br />

CO 2<br />

O H<br />

<strong>Chapter</strong> 16<br />

NH 3<br />

O 2C N CO 2<br />

A<br />

H<br />

CO 2<br />

O 2C N H<br />

fig 3<br />

P16.8: This reaction is analogous to that catalyzed by the pyruvate dehydrogenase<br />

complex (section 16.12B). The additional c<strong>of</strong>actors are thiamine diphosphate, lipoamide,<br />

and FAD. (McMurry p. 186 step 4)<br />

P16.9: (McMurry p. 257)<br />

CoAS<br />

O<br />

O<br />

SCoA<br />

CO 2<br />

H 2O<br />

OH O H2O HSCoA<br />

CoAS<br />

SCoA<br />

NADH<br />

A<br />

H<br />

NAD +<br />

CoASH<br />

OH<br />

H<br />

O H<br />

C<br />

B<br />

fig 4<br />

P16.10: See Biochemistry 39, 6732 for the original report on this experiment.<br />

a)<br />

O<br />

O<br />

O<br />

O<br />

O<br />

A<br />

O<br />

O<br />

:B<br />

:B<br />

NAD +<br />

NADH<br />

O<br />

NH 3<br />

CO 2<br />

185


186<br />

H H<br />

S S<br />

DsbA<br />

S S<br />

DsbA<br />

:B<br />

S S<br />

DsbB<br />

H H<br />

S S<br />

DsbB<br />

H A<br />

B:<br />

A<br />

H<br />

H S<br />

S S<br />

DsbA<br />

DsbB<br />

SH<br />

<strong>Chapter</strong> 16<br />

fig 4<br />

b) The bromoalanine side chain prevents formation <strong>of</strong> the key disulfide bond in DsbB,<br />

and provides an alternative carbon electrophile for the cysteine in DsbA to attack. The<br />

result <strong>of</strong> this SN2 displacement is a stable sulfide linkage beetween the two proteins, and<br />

isolation <strong>of</strong> this species provides evidence for the existance <strong>of</strong> the unstable disulfidelinked<br />

DsbA-DsbB intermediate in the normal reaction.<br />

H H<br />

S S<br />

DsbA<br />

:B<br />

Br<br />

DsbB<br />

SH<br />

proteins linked by<br />

sulfide bond<br />

H<br />

S S<br />

DsbA<br />

DsbB<br />

fig 4<br />

P16.11 This reaction is an electrophilic aromatic substitution, with the flavin peroxide<br />

oxygen as the electrophile. The substitution takes place ortho to the electron-donating,<br />

ortho/para-directing exocyclic amine, and meta to the electron-withdrawing, metadirecting<br />

carbonyl. McMurry p. 247<br />

P16.12: (See also J. Am. Chem. Soc. 2004, 126, 15060; J. Nat. Prod. 2001, 64, 597).<br />

fig 5<br />

P16.13:<br />

H 3C<br />

CH 2<br />

CH3 OH<br />

Br<br />

CH 3<br />

H 3C<br />

CH 2<br />

CH3 OH<br />

Cl:<br />

Br<br />

CH 3<br />

H 3C<br />

SH<br />

CH 2<br />

CH3 OH<br />

Br<br />

CH 3<br />

Cl


fig 5<br />

P16.14:<br />

HO<br />

H 3C COOCH3<br />

O<br />

O<br />

O OH<br />

OH OH<br />

A H<br />

H SG<br />

a) b)<br />

c)<br />

d)<br />

BrMg<br />

P16.15:<br />

a)<br />

O<br />

COOCH 3<br />

O<br />

A<br />

O<br />

O<br />

Br<br />

:B<br />

O<br />

A O<br />

B<br />

OH O<br />

C<br />

O<br />

O<br />

HO<br />

O<br />

O<br />

O OH<br />

stable hemiacetal<br />

HO<br />

O<br />

HO<br />

A H<br />

B<br />

O<br />

O<br />

O<br />

HO OH<br />

OH<br />

O<br />

O<br />

SG<br />

OH OH<br />

OH<br />

+ GSSG<br />

OH<br />

H SG<br />

:B<br />

<strong>Chapter</strong> 16<br />

187


fig 6<br />

b)<br />

fig 7<br />

c)<br />

188<br />

H 3C<br />

P16.16:<br />

OH<br />

O<br />

O<br />

O<br />

FeCl 3<br />

+<br />

Cl<br />

Na<br />

H 2<br />

Pd/CaCO 3<br />

NaNH 2<br />

O<br />

PCC<br />

(CH 3CH 2) 3N<br />

Zn(Hg)<br />

H 3C<br />

H 3O +<br />

Cl<br />

+_<br />

O<br />

HCl<br />

O<br />

DMSO<br />

Cl<br />

O<br />

Cl<br />

FeCl 3<br />

OH<br />

Pd/CaCO 3<br />

O<br />

OOH<br />

HO OCH 3<br />

O<br />

H 3C Cl<br />

H 2<br />

OH<br />

H 3O +<br />

O<br />

NaOCH 3<br />

<strong>Chapter</strong> 16


a)<br />

R OH<br />

b)<br />

H 3C S CH 3<br />

O<br />

R OH<br />

Cl<br />

O<br />

Cr<br />

O<br />

Cl<br />

Cl<br />

H<br />

R O Cr<br />

H<br />

O R<br />

H 3C S CH 3<br />

O<br />

O<br />

O<br />

H<br />

R O Cr<br />

H<br />

R O<br />

H<br />

H<br />

H<br />

O R<br />

H 3C S CH 3<br />

Cl<br />

O<br />

O<br />

OH<br />

H<br />

O<br />

R<br />

:B<br />

<strong>Chapter</strong> 16<br />

189


In-chapter exercises<br />

E17.1:<br />

fig 1a<br />

E17.2:<br />

Cl<br />

Cl<br />

fig 1a<br />

E17.3:<br />

190<br />

H<br />

Cl<br />

Cl<br />

Cl<br />

<strong>Chapter</strong> 17<br />

<strong>Chapter</strong> 17 <strong>solutions</strong><br />

O O O O<br />

Cl<br />

C C<br />

Cl<br />

C<br />

Cl Cl<br />

H<br />

H Cl<br />

X<br />

Cl<br />

Cl<br />

Cl<br />

Cl<br />

H<br />

Cl<br />

Cl<br />

H<br />

C C<br />

X<br />

H<br />

Cl<br />

H<br />

C C<br />

H<br />

H Cl<br />

Cl<br />

Cl<br />

H<br />

Cl<br />

Cl<br />

Cl<br />

Cl<br />

H<br />

C C<br />

X<br />

H<br />

Cl<br />

Cl<br />

C<br />

Cl Cl<br />

H<br />

C C<br />

H<br />

H<br />

Cl<br />

Cl<br />

Cl<br />

Cl<br />

Cl<br />

etc<br />

Cl<br />

Cl


End-<strong>of</strong>-chapter problems<br />

P17.1:<br />

To simplify matters, we'll use 'R' to abbreviate the methyl ester group:<br />

H 2C<br />

CH 3<br />

O<br />

O<br />

CH 3<br />

R<br />

=<br />

CH 3<br />

H 2C C R<br />

The first two propagation steps, forming a acrylamide dimer, are shown below:<br />

X<br />

CH 3<br />

The polymer is represented by:<br />

P17.2:<br />

a)<br />

O<br />

O S O O S<br />

O<br />

O<br />

O<br />

R<br />

2<br />

CH 3<br />

=<br />

X R<br />

R<br />

H<br />

etc.<br />

C C<br />

H<br />

CH 3<br />

C<br />

O<br />

OCH 3<br />

O<br />

O O S O<br />

O<br />

n<br />

X<br />

CH 3<br />

R<br />

CH 3<br />

R<br />

CH 3<br />

R<br />

CH 3<br />

<strong>Chapter</strong> 17<br />

191


192<br />

O<br />

O S O<br />

O<br />

O<br />

2- O4S NH 2<br />

O<br />

O<br />

NH 2<br />

NH 2<br />

2- O4S<br />

O<br />

2- O4S NH 2<br />

O<br />

O<br />

NH 2<br />

n<br />

O<br />

NH 2<br />

polyacrylamide<br />

NH 2<br />

<strong>Chapter</strong> 17<br />

b) The bis-acylamide molecule has two alkene groups, at either end, that can participate<br />

in radical chain elongation reactions. This allows it to tie two growing polyacrylamide<br />

strands together (ie to form a cross-link):<br />

strand 1<br />

strand 1<br />

O<br />

N<br />

N<br />

H H<br />

O<br />

strand 2<br />

strand 2<br />

Here is a more detailed mechanism. We start with a growing polyacrylamide strand<br />

(strand 1) reacting in a chain propagation reaction with a bis-acrylamide molecule. In the<br />

next, step, the remaining alkene group on bis-acrylamide reacts adds to a second growing<br />

polyacrylamide strand (strand 2).


strand 1<br />

strand 1<br />

strand 1<br />

O<br />

strand 1 contiues to<br />

grow from this point<br />

strand 1<br />

O<br />

strand 1<br />

O<br />

O<br />

NH 2<br />

O<br />

O<br />

NH 2<br />

NH 2<br />

O<br />

NH 2<br />

N<br />

N<br />

H H<br />

O<br />

NH 2<br />

O<br />

NH 2<br />

N<br />

O<br />

N<br />

NH 2<br />

O<br />

O<br />

N<br />

H H<br />

N<br />

H H<br />

O<br />

N<br />

N<br />

H H<br />

O<br />

O<br />

H 2N O<br />

H 2N O<br />

O<br />

H 2N O<br />

O<br />

NH 2<br />

NH 2<br />

strand 2<br />

strand 2<br />

strand 2<br />

strand 2 continues to<br />

grow from this point<br />

<strong>Chapter</strong> 17<br />

193


P17.3:<br />

194<br />

HO<br />

OH<br />

O<br />

<strong>Chapter</strong> 17<br />

Notice that the radical form <strong>of</strong> resveratrol shown above is more stable, compared to a<br />

radical species in which the unpaired electron is located on one <strong>of</strong> the 'lower' phenoxy<br />

groups. The extra stability is due to resonance - the unpaired electron can be delocalized<br />

over both <strong>of</strong> the aromatic rings. (Consider the two other alternative radical species<br />

below - in these cases, the unpaired electron cannot be delocalized over both rings! It all<br />

comes back to para vs. meta positioning on the aromatic ring.)<br />

P17.4:<br />

P17.5:<br />

H 3C<br />

CH 3<br />

C<br />

CN<br />

O<br />

N N<br />

OH<br />

CH 3<br />

C<br />

CN<br />

O<br />

OH<br />

OH<br />

HO<br />

a) b)<br />

c) d)<br />

CH 3<br />

HO<br />

CH3 H3C C N N<br />

CN<br />

HO<br />

O<br />

N 2 gas<br />

OH<br />

CH 3<br />

C<br />

CN<br />

S<br />

CH 3<br />

OH<br />

O


<strong>Chapter</strong> 17<br />

The driving force for homolytic cleavage is the formation <strong>of</strong> nitrogen gas, which is very<br />

entropically favorable.<br />

P17.6:<br />

a) Ege p. 770<br />

Cl Cl 2 Cl<br />

H<br />

H 3C C<br />

H<br />

H<br />

H Cl H 3C C<br />

H<br />

H 3C C<br />

+ H Cl<br />

b) Butane could form from an alternate chain termination step:<br />

c)<br />

H<br />

H 3C C<br />

H<br />

H<br />

C<br />

H<br />

H<br />

H<br />

Cl<br />

CH 3<br />

Cl<br />

H<br />

H 3C C C<br />

H<br />

H<br />

H<br />

CH 3<br />

195


196<br />

H 3C<br />

Cl<br />

H 3C<br />

H 2C<br />

H<br />

C C<br />

CH 2<br />

H<br />

H<br />

H<br />

CH 3<br />

H 3C<br />

H 3C<br />

H 3C<br />

H<br />

C C<br />

H<br />

H 3C<br />

H<br />

C C<br />

CH 3<br />

H<br />

H<br />

CH 3<br />

H 3C<br />

+ Cl<br />

CH 3<br />

C C<br />

H<br />

H<br />

CH 3<br />

H 3C<br />

H 3C<br />

H<br />

C C<br />

A B C D<br />

C C<br />

H<br />

H<br />

CH 3<br />

H 3C<br />

H 3C<br />

C C<br />

Cl<br />

H<br />

H<br />

CH 3<br />

H 3C<br />

H 3C<br />

C C<br />

H<br />

H<br />

CH 3<br />

H 3C<br />

H 3C<br />

H<br />

C C<br />

<strong>Chapter</strong> 17<br />

d,e) Radical intermediate B is the most stable (it is tertiary), but the resulting alkyl<br />

chloride is not necessarily the most abundant product. Even though radical A is less<br />

stable than radical B, there are six possible pathways for its formation (the chlorine<br />

radical could abstract six different hydrogens to form radical A), as opposed to only one<br />

pathway for the formation <strong>of</strong> B. There are two pathways to radical C, and three to D.<br />

P17.7:<br />

a) The peroxide contaminant acts as a radical chain initiator:Vollhardt 5th p. 531<br />

RO OR 2 RO<br />

The regioselectivity arises from the higher stability <strong>of</strong> the secondary alkyl radical<br />

intermediate (compared to the alternate possibility, which would be a primary radical).<br />

RO<br />

H Br ROH + Br<br />

Cl<br />

Br<br />

H Br<br />

H<br />

H<br />

H<br />

H<br />

+ Br<br />

CH 2<br />

CH 2<br />

Cl<br />

Br


)<br />

RO<br />

H SCH 2CH 3 SCH 3CH 3<br />

P17.8: (McMurry BA p. 146)<br />

O<br />

O<br />

H<br />

Fe<br />

O<br />

H<br />

R 1<br />

R 2<br />

CO 2<br />

O<br />

O<br />

H<br />

S<br />

H SCH 2CH 3<br />

R 1<br />

R 2<br />

R 1<br />

R 2<br />

O<br />

O<br />

S<br />

+<br />

SHCH2CH3 <strong>Chapter</strong> 17<br />

P17.9: Silverman p. 204, J. <strong>of</strong> Pharmacol. Exp. Therapeutics Fast Forward 2007, 321,<br />

590<br />

a)<br />

O<br />

O<br />

H<br />

H<br />

R 1<br />

R 2<br />

197<br />

R 1<br />

R 2


)<br />

198<br />

N<br />

N<br />

O<br />

Fe<br />

S<br />

enz<br />

HO H<br />

N<br />

N<br />

N<br />

O<br />

HO H<br />

N<br />

O<br />

H<br />

C<br />

H<br />

H<br />

N<br />

N<br />

P17.10: (xxSilverman p. 222)<br />

O<br />

Fe<br />

S<br />

enz<br />

N<br />

N<br />

N<br />

N<br />

O<br />

Fe<br />

S<br />

enz<br />

N<br />

N<br />

R 1<br />

R 2<br />

<strong>Chapter</strong> 17<br />

N<br />

N<br />

HO H<br />

N<br />

O<br />

CH 2<br />

N<br />

N<br />

HO H<br />

N<br />

N<br />

N<br />

O<br />

Fe<br />

S<br />

enz<br />

O<br />

Fe<br />

S<br />

enz<br />

OH<br />

Fe<br />

S<br />

enz<br />

OH<br />

N<br />

N<br />

R 1<br />

R 2<br />

N<br />

N<br />

N<br />

N


HO<br />

HO<br />

P17.11:<br />

fig 7<br />

enz<br />

Cu<br />

O<br />

OH<br />

H<br />

N<br />

N<br />

enz<br />

Cu<br />

OH<br />

O<br />

N<br />

R H<br />

enz<br />

enz<br />

Cu<br />

O<br />

O<br />

H H<br />

N<br />

NH 2<br />

H<br />

O<br />

OH<br />

enz<br />

O O<br />

R<br />

A H<br />

H<br />

N<br />

N<br />

R<br />

R<br />

(semiquinone)<br />

enz<br />

Cu<br />

O<br />

O<br />

N<br />

O<br />

NH 2<br />

enz<br />

O OH<br />

N<br />

NH 2<br />

H<br />

O<br />

H 2O<br />

OH<br />

enz<br />

enz<br />

Cu<br />

O<br />

R<br />

H<br />

N<br />

N<br />

R<br />

H<br />

enz<br />

Cu<br />

O<br />

R<br />

OH<br />

O<br />

O<br />

N<br />

<strong>Chapter</strong> 17<br />

O<br />

NH 2<br />

N<br />

enz<br />

H<br />

O<br />

OH<br />

NH 2<br />

enz<br />

199

Hooray! Your file is uploaded and ready to be published.

Saved successfully!

Ooh no, something went wrong!